86% found this document useful (7 votes)
7K views

Pediatrics SLE MCQ From 2018-2014

1. The mother would be given phenobarbital to take home in case of another seizure in her baby with tonic clonic convulsions. 2. The 4 week old male child with acute onset forceful non-bilious vomiting after feeding and normal weight gain would be diagnosed with pyloric stenosis. 3. The initial treatment for a child presenting with stiff neck, fever, and headache suspected of having meningitis would be penicillin. 4. It is recommended to give Vitamin K shortly at birth and Vitamin D at 2 months for breastfeeding neonates.

Uploaded by

Asif Newaz
Copyright
© © All Rights Reserved
Available Formats
Download as DOCX, PDF, TXT or read online on Scribd
86% found this document useful (7 votes)
7K views

Pediatrics SLE MCQ From 2018-2014

1. The mother would be given phenobarbital to take home in case of another seizure in her baby with tonic clonic convulsions. 2. The 4 week old male child with acute onset forceful non-bilious vomiting after feeding and normal weight gain would be diagnosed with pyloric stenosis. 3. The initial treatment for a child presenting with stiff neck, fever, and headache suspected of having meningitis would be penicillin. 4. It is recommended to give Vitamin K shortly at birth and Vitamin D at 2 months for breastfeeding neonates.

Uploaded by

Asif Newaz
Copyright
© © All Rights Reserved
Available Formats
Download as DOCX, PDF, TXT or read online on Scribd
You are on page 1/ 61

1.

Baby with tonic clonic convulsions, what drug you'll give the mother to take home if there is
another seizure?
a) Diazepam
b) phenytoin
c) phenobarbital

2. 4 weeks old male child with acute onset forceful non-bilious vomiting after feeding. He is the first child in
the family. He is gaining normal weight and looks hungry. What’s your diagnosis?
a) Pyloric stenosis

3. Child present with stiffing neck, fever, headache. You suspect meningitis what is your initial treatment?
a) Tobramycin
b) Levoflaxicine
c) Penicillin
d) Doxycycline

4. Breast feeding in the full term neonate:


a) Increase URTI rate
b) No need for vitamin supplementation
c) Food introduce at 3 months
d) Increase GE rate
e) It’s recommended to give Vitamin K shortly at birth & Vitamin D at 2 months.

5. 9 days old neonate is brought by his mother for check-up. He was delivered by spontaneous normal
vaginal delivery without complications. Birth weight was 3.4 and his birth weight now 3.9. He is sucking
well and looks normal except for jaundice. What’s your diagnosis?
a) Physiological jaundice
b) Breast milk jaundice
c) Crigler-Najjar syndrome
d) ABO incompatibility

6. A full term baby boy brought by his mother weight 3.8 kg. Developed jaundice at 2nd day of life.
Coomb’s test –ve, Hb: 18, bilirubin: 18.9 & indirect: 18.4, O/E: baby was healthy and feeding well, the
most likely diagnosis is
a) Physiological jaundice
b) ABO incompatibility
c) Breast milk jaundice
d) Undiscovered neonatal sepsis
 Breast milk jaundice is a different which tends to develops after the first 4-7 days of life, continues up to
the sixth week of life. Itoccurs early caused by insufficient breast milk intake. (Low calories).
 Physiologic jaundice: manifests after the first 24 hours of life

7. The cardiac arrest in children is uncommon but if occur it will be due to primary
a) Respiratory arrest
b) hypovolemic shock
c) Neurogenic shock

8. After doing CPR on child and the showing asystole:


a) Atropine
b) Adrenaline
c) Lidocaine

298
9. A baby came complaining of croup, coryza, air trapping, tachypnea & retraction. The best management is:
a) Erythromycin
b) Penicillin
c) Ampicillin
 Since croup is usually a viral disease, antibiotics are not used unless secondary bacterial infection
is suspected.
 In cases of possible secondary bacterial infection, the antibiotics vancomycin and cefotaxime are
recommended.
 In severe cases associated with influenza A or B, the antiviral neuraminidase inhibitors may be
administered.

10. 5 years old boy brought to the ER by his mother complaining of drooling saliva, inability to drink &
eat. On examination there was a congested larynx. The most appropriate diagnosis is:
a) Viral pneumonia
b) Croups
c) Acute epiglottitis
d) Bacterial pneumonia
e) Bronchiolithitis
 It occurs at any age, rapid onset, causes drooling of saliva & inability to drink or eat, no cough &
you could see the congested larynx.
 Croup has a slow onset, occurs at ages <4 years with a barking cough & the ability to swallow fluids

11. 15 years old boy had history of URTI 2 weeks ago. Now he is complaining of fever, bilateral knee pain with
swelling & tenderness. The diagnosis is:
a) Sickle cell anemia
b) Post-streptococcal Glomerulonephritis
c)Rheumatoid arthritis (JRA)
d) Rheumatic fever
e) Septic arthritis

12. 10 years old boy presented with a 5 days history of skin lesion which was scaly & yellowish. What is the
diagnosis?
a) Tinea corporis

13. Apgar score


a) Heart rate is an important criterion.
b) Is out of 12 points.
c) Gives idea about favorability of vaginal delivery.
d) Taken at delivery time and repeated after 5 minutes.
e) Respiratory rate is an important criterion
 A,D&E are correct
 This scoring system provided a standardized assessment for infants after delivery. The Apgar score
comprises 5 components: heart rate, respiratory effort, muscle tone, reflex irritability, and color, each
of which is given a score of 0, 1, or 2. the score is now reported at 1 and 5 minutes after birth.

14. Dehydration 25 kg kid, maintenance


a) 1200
b) 1300
c) 1400
d) 1500 (1600)
15. 11 months old baby, 10 kgs, maintenance daily fluid :
a) 1000 ml
b) 500 ml
c) 2000 ml
d) 2500 ml

16. Mother has baby with cleft palate and asks you what is the chance of having a second baby with cleft
palate or cleft lip:
a) 25%
b) 50%
c) 1 %
d) 4%

17. 10 years old child with rheumatic fever treated early, no cardiac complication. Best to advice the
family to continue prophylaxis for:
a) 5 months
b) 3 years
c) 6 years
d) 15 years
 Duration of Secondary Prophylaxis for Rheumatic Fever

Type Duration after last attack

0 years or until age 40 years (whichever is residual heart disease (persistent longer); lifetime prophylaxis may be needed va

Rheumatic fever with carditis but


no residual heart disease (no 10 years or until age 21 years (whichever is
valvular disease) longer)
Rheumatic fever without carditis5 years or until age 21 years (whichever is longer)

18. Hematological disease occurs in children, treated with heparin and fresh frozen plasma what is
the disease?
a) Hemophilia A
b) Hemophilia B
c) Von-wille brand disease
d) DIC thrombosis

19. Child is complaining of severe headache which is unilateral, throbbing and aggravated by light, diagnosis:
a) Migraine
b) Cluster Headache
c) Stress Headache

20. Infant with features of Down syndrome, the most likely this infant has
a) Trisomy 21

311
21. 8 months old infant with on & off recurrent crying episodes & history of currant jelly stools:
a) Intussception
b) Intestinal obstruction
c) Mickel's diverticulitis
d) Strangulated hernia

22. Baby with crying episodes and currant jelly stool, looks slightly pale, signs of obstruction what is
the management?
a) Barium enema
b) immediate surgery
c) IV fluid & wait for resolution

23. Baby having HIV (transmitted from his mother), which vaccination shouldn't be given to him?
a) Oral polio
b) MMR

 Live attenuated vaccines (oral polio, MMR, rotavirus, varicella, BCG) are contraindicated in patients with
immunodeficiency.

24. Most common chromosomal abnormality:


a) Down's syndrome (trisomy 21)
b) Turner's syndrome
c) Klienfilter's syndrome

25. Who should not get the oral polio vaccine?

 OPV should not be given when there is a higher risk of bad effects caused by the vaccine, including the
following:
 Being moderately or severely (badly) ill with or without fever.
 Having someone in the house with a weak immune system
 History of a severe allergic reaction to a dose of OPV
 Long-term treatment with steroid medicine.
 Weak immune system. The immune system is the part of the body that normally fights off sickness and
disease. A weak immune system may be caused by cancer, HIV or AIDS, inborn immune deficiency, or
taking medicines, such as chemotherapy.

26. Mother brought her 18-month-old infant to ER with history of URTI for the last 2 days with mild
respiratory distress. This evening the infant start to have hard barking cough with respiratory
distress. O/E: RR 40/min, associated with nasal flaring, suprasternal & intercostal recessions. What is
the most likely diagnosis?
a) Viral Pneumonia
b) Bacterial Pneumonia
c) Bronchiolitis
d) Acute epiglottitis
e) Trachiobronchiolitis

27. Intellectual ability of child measured by


a) CNS examination
28. A child swallowed his relative's medication. What is the best way of gastric decontamination?
a) Gastric lavage
b) Total bowel irrigation (whole bowel wash)
c) Syrup ipecac
d) Activated charcoal

29. Infant swallow cohesive material came within half an hour to ER drooling, crying. what is the initial
thing to do?
a) activated charcoal
b) endoscopy
c) secure airway
d) 2 cups of milk

30. Child ate overdose of iron, best immediate management:


a) Gastric lavage ( because immediate )
b) Induce vomiting manually
c) Emetic drugs
d) Ipecac
e) IV Deferoxamine

31. Child has pallor, eats little meat, by investigation microcytic hypochromic anemia, what will you do?
a) Trial of iron therapy
b) Multivitamin with iron daily

32. Child came with fatigue 'pic of anemia' and stunted growth, his blood works shows microcytic
hypochromic anemia, diagnosis is:
a) Thalassemia
b) Sideroblastic
c) lead poisoning
d) Iron deficiency anemia
e) SCA

33. Female her height is 10th percentile of population, what u will tell her about when spinal length
completed, after menarche?
a) 6m
b) 12 m
c) 24 m
d) 36 m

34. 6 years old with HBsAg his mother has HBV he did not receive any vaccination except BCG he should take:
a) DT, Hib,MMR,OPV
b) DTB,Hib,MMR,HBV,OPV
c) DTB, Hib, MMR, OPV
d) Td, Hib,MMR,OPV,HBV
e) TDap, MMR, IPV, HBV

35. Infant with bright blood, black stool and foul smelling stool. Best way to know the diagnosis:
a) US
b) Radio Isotope scan
c) Angiogram

312
36. 3 days old baby, his mother HBV positive, what is your action?
a) one dose immunoglobulin and vaccination
b) immunoglobulin
c) three doses HBV vaccine

 Infant of mother HBV-positive must receive immunoglobulin within first 12 hour and vaccination as
0,1 and 6 months For this child it is too late for immunoglobulin

37. 2 month infant with vomiting after each meal, he is in 50 centile, He passed meconium early and stool,
diagnosis is:
a) Midgut volvulus
b) Meconium ileus
c) Hischsprung disease

38. Child was sick 5 days ago culture taken showed positive for meningococcal. Patient now at home
and asymptomatic your action will be:
a) Rifampicin
b) IM ceftriaxone

39. What is the injection that is routinely given to new-born to inhibit haemorrhage:
a) Vitamin K
b) Vitamin C
c) Vitamin D
d) Vitamin E

40. Child with URTI is complaining of bleeding from nose, gum and bruising the diagnosis is:
a) Hemophilia A
b) ITP

41. Child came with his father and has high BMI and look older than other children with same age, on
exam child has >95th percentile of weight and tall, management is:
a) Observe and appoint
b) Life style change
c) Give program to decrease the weight

42. Newborn came with red-lump on left shoulder, it is:


a) Haemangioma

43. 3 months infant with red swelling that increase in size rapidly:
a) Pot-wine spot
b) Cavernous hemangioma

44. Newborn came with congenital hepatomegaly, high LFT, jaundice the most organism cause this symptoms
is:
a) Congenital TB
b) Rubella
c) HIV
d) CMV
45. One of the following is NOT a feature of Henoch-Schonlein purpura (HSP):
a) Arthritis.
b) Rash over the face.
c) Abdominal pain.
d) Normal platelet count.

46. Child with URTI what is the most helpfully sign that it is viral
a) Colorless nose discharge

47. Child with picture of SCA he should be maintained on:


a) Penicillin and folic acid

48. After bite, pediatric patient presented with abdominal pain and vomiting , stool occult blood, rash over
buttock and lower limbs, edema of hands and soles, urine function was normal but microscopic
hematuria was seen:
a) Lyme
b) Henoch-Schonlein Purpura

49. Child develop purpuric rash over his extremities, this rash was preceded by upper respiratory
tract infection 1 week ago. What is your diagnosis?
a) ITP
b) Henoch-Schonlein purpura
 HSP skin rash distribution: lower extremities (dorsal surface of the legs), buttocks, ulnar side of arms
& elbows
 Workup: CBC: can show leukocytosis with eosinophilia & a left shift, thrombocytosis in 67% of cases.
 Decreased platelets suggest thrombocytopenic purpura rather than HSP.

50. Cellulitis in neonate mostly caused by


a) Group B streptococcus

51. Henosch-Scholein purpura affect:


a) Capillary
b) Capillary and venule
c) Arteriole, capillary and venule
d) Artery to vein

52. Baby with conjugated hyperbilirubinemia:


a) Biliary atresia
b) ABO comp
c) G6PD

53. Child with DM came with picture of DKA, which HLA is responsible?
a) DR4
b) DR5
c) DR6
d) DR7
e) DR3 and DR4
54. 7 years old child had history of chest infection which was treated with antibiotics. The patient presented
6 weeks after cessation of antibiotics with abdominal pain, fever and profuse watery diarrhea for the past
month. Which of the following organisms is responsible for the patient’s condition?
a) Giardia Lamblia
b) Clostridium Difficile
c) Escherichia coli
d) Clostridium Perfringens

 Causes severe diarrhea when competing bacteria in the gut flora have been wiped out by antibiotics.

55. For the above disorder, which one is considered pathological?


a) Gross hematuria
b) Microscopic hematuria
c) Rashes

56. Gross motor assessment at age of 6 months to be asked is:


a) Sitting without support
b) Standing
c) Role from prone to supine position
d) Role from supine to prone position

57. The immediate urgent referral of child that take?


a) 10 pills contraceptive
b) 10 pills antibiotics
c) 75 mg Paracetamol

58. Child woke up with croup, what should you put in your DDx?
a) Pneumonia
b) Tonsillitis
c) Foreign body

59. Child came with wheezing and cough and diagnosed with asthma and his dr. prescribe to him
beclomethasone space inhaler or nebulizer am not sure twice daily… what most worried side
effect of using it:
a) Growth retardation
b) Extaoriculer problem

60. Twins (boy and girl) the father came asking why his daughter start puberty before his son:
a) Girls enter puberty 6-12 months before boys
b) Girls enter puberty 2-3 years before boys
c) Girls enter puberty 1-2 years after boys
d) Girls enter puberty as the same age of boys

61. A boy with nocturnal enuresis, psychotherapy failed to show result you will start with:
a) Imipramine and vasopressin
b) clonidine and vasopressin
c) clonidine and guanfacine
d) Imipramine and guafacine
62. Earlier sign of puberty in male is:
a) Appearance of pubic hair
b) Increase testicular size
c) Increase penis size
d) Increase prostate size
 The first sign of puberty in boys in testicular enlargement more than 2.5 centimeters which followed
by a growth spurt 1-2 years later and beginning of spermatogenesis.

63. Boy came with history of wheel on erythematous base after 10 day you find in the examination preorbital
swelling, supraclavicular L.N., hepatomegaly and splenomegaly what is the diagnosis?
a) Angioedema
b) Urticarial
c) Lymphoma

64. Male patient with a cyanotic heart disease, all except:


a) ASD
b) VSD
c) PDA
d) Truncus arteriosus

65. Cellulitis in children most common causes:


a) Group A streptococcus
b) Staphylococcal aureus

 Staphylococcus aureus is the most common bacteria that cause cellulitis.


 Group A Streptococcus is the next most common bacteria that cause cellulitis. A form of rather
superficial cellulitis caused by strep bacteria is called erysipelas; it is characterized by spreading hot,
bright red circumscribed area on the skin with a sharp raised border. The so-called “flesh-eating
bacteria” are, in fact, also a strain of strep which can – in severe cases – destroy tissue
almost as fast as surgeons can cut it out.

66. In developing countries to prevent dental caries, it add to water:


a) Flouride
b) Zink
c) Copper
d) iodide

67. Baby complaining of fever , chills , rigors and neck rigidity +ve kurnings sign rash on his lower
limb diagnosis:-
a) Meningeococcal meningitis

68. 6 months baby with undescended testis which is true:


a) Till the mother that he need surgery
b) In most of the cases spontaneous descent after 1 year
c) Surgery indicated when he is 4 years
d) Unlikely to become malignant

 Explanation: Most of spontaneous descent cases occur before the age of 6 months. If no descent
occurs at 6 months, surgery is indicated before the age of 2 years.
69. Forcing the child to go to the toilet before bedtime and in the morning, you’ll control the problem
of;
a) Enuresis

70. Cellulitis occurring about the face in young children (6-24 months) and associated with fever and purple
skin discoloration is MOST often caused by
a) group A beta hemolytic streptococci
b) Haemophilus influenzae type B
c) streptococcus pneumonie
d) staphylococcus aureus
e) pseudomonas
 Facial cellulitis includes both When associated with trauma or contiguous infection (eg, stye),
Staphylococcus aurous or Streptococcus pyogenes are likely causes
 In the absence of trauma or contiguous infection, historically Haemophilus influenzae type b was the
most common cause followed by Streptococcus pneumonia

72. Patient with atopic dermatitis and he is 2 years old came with cough and wheezing:
a) bronchiolitis
b) Bronchial asthma

73. Child known case of sickle cell disease with recurrent UTI which is treated, Now he is stable (cbc,chem.
within normal) you can discharge him with:
a) Prophylactic Penicillin
b)Iron

74. 2 years old known case of sickle cell disease child with hand and foot swelling, crying, You will
discharge him with:
a) Penicillin and vaccination

75. Most common malignancy in children


a) Leukemia
b) wilms tumor

 Wilms tumor: The most common feature at presentation is an abdominal mass. Abdominal pain
occurs in 30%-40% of cases. Other signs and symptoms of Wilms tumor include hypertension,
fever caused by tumor necrosis, hematuria, and anemia.
 A renal tumor of embryonal origin that is most commonly seen in children 2–5 years of
age. Associated with Beckwith-Wiedemann syndrome (hemihy-pertrophy, macroglossia,
and visceromegaly), neurofi bromatosis, and WAGR syndrome (Wilms’, Aniridia,
Genitourinary abnormalities, mental Retardation).
 Presents as an asymptomatic, nontender, smooth abdominal mass, abdominal pain,
fever, hypertension, and microscopic or gross hematuria.
 Treatment : Local resection and nephrectomy with postsurgical

76. Mild diarrhoea management:


a) ORS

77. Most common intra-abdominal tumour in children:


a) Wilm's tumor
b) Lymphoma
78. Diagnosis of thalassemia minor:
a) HB a2 and HB f
b) Microcytosis
 Beta Thalassemia Minor:
 The thalassemia seen most commonly is caucasians (primarily Mediterranean descent)
 Beta thalassemia minor is loss of one of two genes for Beta globin on chromosome 11
 Patients generally asymptomatic
 May have mild microcytic anemia (MCV: 60-70; Hgb: 10-13) with a normal or slightly increased RBC
count
 The peripheral smear will show target cells and basophilic stippling
 See increased HbA2 in the range of 5-9% with normal HbF
 Diagnosis may be obscured in concomitant iron deficiency present because Beta-thalassemia causes
an increase in HbA2 while iron deficiency causes a decrease in HbA2. Both create a microcytosis.
 Beta Thalassemia Major:
 Homozygous double gene deletion with no Beta globin production
 Presents with lethal anemia, jaundice, splenomegaly, growth retardation, bone malformations,
death
 Severe hypochromic, microcytic anemia with very bizarre cells
 HbA2 is not increased
 HgF is at nearly 100%

79. Celiac disease which not cause it:


a) Rice & corn
b) Oat
c) wheel
d) Gluten

80. Celiac disease involves :


a) Proximal part of small intestine
b) Distal part of small intestine
c) Proximal part of large intestine
d) Distal part of large intestine

81. 4 years old baby comatose and cyanotic in the kitchen, there was peanuts in his hand:
a) Aspiration

82. 15 years old boy with unilateral gynecomastia your advice is


a) May resolve spontaneously
b) There is variation from person to person
c) Decrease use of soda oil or fish oil
 Uni- or bilateral gynecomastia occur normally in newborn & at puberty

83. 6 months old boy with fever you should give antipyretic to decrease risk of
a) Febrile convulsion
b) Epilepsy
c) Disseminate bacteria

84. 6 years old with cyanosis, at 6 months similar attack, what is best investigation?
a) Pulmonary function test
85. 6 months old with cough and wheezy chest .diagnosis is:
a) asthma (after 2 years old)
b) Bronchiolitis (before 2 years old)
c) pneumonia (associated with crypitation)
d) F.B aspiration (sudden wheezing)

86. Child presented with anemia he has family history of thalassemia what the most diagnostic test
a) measuring of HB A2
b) Bone marrow
c) Serum ferritin

 The most diagnostic test is hemoglobin electrophoresis

87. Child presented to ER with SOB on x-ray there is filtration on mid & lower zone on right side after 24h of
antibiotic patient become cyanosis the x-ray total lung collapse with mediastinal shift. what cause
a) H-influenza
b) Pneumocystis carnia
c) Streptococcus pneumonia

88. 8 month boy presented with fever, SOB, poor feeding and confusion. On exam ear was red and
ESR high, what is the next best step in diagnosis?
a) Blood culture
b) CSF
c) Chest X-ray
d) Urine analysis
e) CBC and differential

89. Infant with coryza, wheezing and URTI symptoms came to ER with SOB, what is the first management?
a) Bronchodilator
b) Corticosteroid
c) Theophylline

90. Boy 12 years old came to you complaining of that he worries about himself because he see that his
friends has axillary hair and he is not like them, about sexual maturity of boys what is first feature :
a) Testicular enlargement, in females breast buds
b) penile elongation
c) Hair in axilla
d) Hair in the pubic area

91. Child brought by mother due to bleeding per nose , by examination you found many bruises in his
body, over his back ,abdomen and thigh , what is your diagnosis :
a) Child abuse

92. Child 9 months with congenital heart disease, central and peripheral cyanosis Dx?
a) Tetralogy of fallot
b) Coarctation of aorta
c) Truncus arteriosus
d) ASD
e) PDA
93. Child with spontaneous epistaxis. DX :
a) Coagulation deficiency

94. Three years child presents with diarrhea with blood & mucus for 10 days on investigation no cyst in
stool examination, what is the most common cause?
a) Ulcerative colitis
b) Giardiasis
c) Rota virus
 Bloody diarrhea is a common problem in children.
 Bacterial infections and parasitic infestations are responsible for most of the cases.
 Milk allergy is a frequent cause in young infants.
 Chronic inflammatory bowel disease occurs in older children.

95. Child with dry cough & wheeze, CXR showed hyper-inflated lung with some infiltrate:
a) Bronchial asthma
b) Bronchiolitis.

96. Child anaemic, abdominal pain, blood in faces ( I forget colour of stool & rest of case but I think it
is about volvulus?) next investigation:
a) Abdominal ultrasound.
 Diagnosis of Malrotation with Volvulus :
 AXR may reveal the absence of intestinal gas but may also be normal.
 If the patient is stable, an upper GI is the study of choice and shows an abnormal location of the
ligament of Treitz. Ultrasound may be used, but sensitivity is determined by the experience of
the ultra-sonographer.

97. Patient known case endocarditis will do dental procedure prophylaxis?


a) 2 g amoxicillin before procedure 1 h
b) 1 g amoxicillin after procedure
c) 2 g clindamycin before procedure 1 h
d) 1 g clindamycin after procedure

98. Cow milk differ from mature human milk that cow milk contain more:
a) More protein
b) More Iron content
c) More calories
d) More fat

human milk Cow


Calories 62 59
Carbohydrate 7 4.8
Protein 1.4 3.3
Fat 4.45 3.8

 All minerals are much more in cow milk than human milk except iron & copper.
 Breast milk contain more Vitamin C & D

311
99. child with congested throat & tonsil with white plaque on erythematous base on tongue & lips ,
also there is gingivitis (Dx.)
a) PHARYNGITIS

100. Patient with Kwashiorkor:


a) High protein & high carbohydrate.
b) High protein & low carbohydrate
c) Low protein & high carbohydrate
d) Low protein & low carbohydrate.

101. Nutritional marasmus on definition:


 Kwashiorkor caused by insufficient protein consumption but with sufficient calorie intake,
distinguishing it from marasmus
 Marasmus is a form of severe protein-energy malnutrition characterized by energy deficiency caused
by inadequate intake of proteins and calories. A child with marasmus looks emaciated. Body weight
may be reduced to less than 80% of the average weight that corresponds to the height. Marasmus
occurrence increases prior to age 1, whereas kwashiorkor occurrence increases after 18 months. It
can be distinguished from kwashiorkor in that kwashiorkor is protein wasting with the presence of
edema. The prognosis is better than it is for kwashiorkor.

102. Most cause of URTI


a) RSV
 Viruses cause most URIs, with rhinovirus, parainfluenza virus, coronavirus, adenovirus,
respiratory syncytial virus,

103. Kawasaki disease associated with:


a) Strawberry tongue
 Kawasaki disease: Multisystem acute Vasculitis that primary affected young children. Fever plus four
or more of the following criteria for diagnosis:
1) fever > 40 C for at least five days
2) Bilateral, non-exudative, painless conjunctivitis
3) Polymorphous rash (primarily truncal)
4) Cervical lymphadenopathy (often painful and unilateral)
5) Diffuse mucous membrane erythema ( strawberry tongue ) , dry red
6) Erythema of palm and sol
7) Other manifestation : gallbladder hydrops, hepatitis, arthritis
 Untreated Kawasaki disease can lead to coronary aneurysms and even MI
 Treatment:
1) High dos ASA ( for fever and inflammation) & IVIG ( to prevent aneurysm )
2) Referral to pediatric cardiologist.

104. Child with skin rash, pericarditis, arthritis dx:


a) Kawasaki

105. Child presented with erythematous pharynx, with cervical lymph nodes and rapid strplysin
test negative and low grade fever with positive EBV. it next step
a) Give antibiotics and anti-pyretic
b) Give anti pyretic and fluids
c) Do culture and sensitivity
106. 2 months old child complaining of spitting of food, abdominal examination soft lax, occult blood –
ve, what you will do?
a) Reassure the parents
b) Abdominal CT

107. Baby with streptococcus pharyngitis start his ttt after two days he improved, Full course of
streptococcus pharyngitis treatment with amoxicillin is
a) 10 days (9-11 days)
b) 7days
c) 14 days
 If group A streptococcus is suspected, begin empiric antibiotic therapy with penicillin × 10 days.
 Cphalosporin, amoxicillin, and azithromycin are alternative options.
 Symptom relief can be attained with fluids, rest, antipyretics and salt-water gargles

108. Newborn with pulse 300 bpm, with normal BP, normal RR, what do you will do for newborn?
a) Cardiac Cardioversion
b) Verapamil
c) Digoxin
d) Diltzam IV
 Treatment of supraventricular tachycardia in asymptomatic patients
 Ice to face and vagal maneuvers
 Adenosine
 Propranolol
 Digoxin
 Procainamide

109. Child with fever first after 2 days he got sore throat white yellow mouth lips lesion on erythematous
base with gingivitis Dx?
a) HSV
b) EBV
c) CMV
d) Adenovirus

110. Child in ER , with dyspnoea , tachypnea , subepiglottic narrowing in x-ray :


 If thumbprint sign: Epiglottis
 If steeple sign: croup

Epiglottis (thumbprint sign) steeple sign : croup


312
111. Child on chemotherapy, he developed septicaemia after introduce IV cannula, what is causative
organisms?
a) Hib
b) Pseudomonas
c) E.coli
d) strept
e) Klebsiella

112. Baby born & discharge with his mother, 3weeks later he started to develop difficulty in breathing &
become cyanotic, what is most likely diagnosis?
a) VSD
b) Hypoplastic left ventricle
c) Coarctaion of aorta
d) Subaortic hypertrophy

113. Attention Deficit Hyperactivity Disorder child what is the manegment?


a) Ecitalpram
b) Atomoxetine
c) Olanzapine
d) Clonazepam
 Combination of medications and behavioral therapy is far superior to just medication treatment
 A class of drugs called psychostimulants is a highly effective treatment for childhood ADHD. These
medicines, including Adderall, Concerta, Daytrana and Ritalin, help children to focus their
thoughts and ignore distractions.
 Another treatment is nonstimulant medication. These medications include Intuniv, Kapvay
and Strattera “atomoxetine”

114. Child with moderate persistent BA On bronchodilator inhaler. Presented with acute
exacerbation what will you add in ttt:
a) Corticosteroid inhaler
b) Ipratropium bromide inhaler

115. What is the best source of iron in a 3 month old infant


a) Beast milk
b) Low fat cow milk.
c) Yellow vegetables.
d) Fruit.
e) Iron fortified cereals.
 Infants absorb 100% of the iron in breast milk (less than 1 mg/L), but cannot absorb all of the iron in
infant formulas.

116. 9 months infant, develop anemia, he start cow milk before 2 months, what is the management?
a) Stop milk
b) Give antihistamine

117. Child starts to smile:


a) At birth
b) 2 months (6 weeks)
c) 1month
118. The child can walk without support in:
a) 6months
b) 9months
c) 15 months
d) 18month
 12 months walk with one hand held, 15 months independently and takes a step up at 18 months.

119. Child recognize 4 colours, 5 words, hops on one foot, consistent with which age:
a) 12 months
b) 24 months
c) 36 months
d) 18 months

120. You received a call from a father who has a son diagnosed recently with DM-I for six months, he said
that he found his son lying down unconscious in his bedroom, What you will tell him if he is seeking
for advice:
a) Bring him as soon as possible to ER
b) Call the ambulance
c) Give him his usual dose of insulin
d) Give him IM Glucagon
e) Give him Sugar in Fluid per oral

121. Cardiac congenital heart disease in children, all true except:


a) 4-5%
b) VSD is the commonest
c) ASD patient should not play a competition.
 Congenital heart defects can be related to an abnormality of an infant's chromosomes (5 to 6
percent), single gene defects (3 to 5 percent), or environmental factors (2 percent). In 85 to 90
percent of cases, there is no identifiable cause for the heart defect, and they are generally considered
to be caused by multifactorial inheritance.

122. 4 years old brought by his parents with weight > 95thpercentile, height < 5th percentile & bowing of
both legs what is the appropriate management?
a) Liver & thyroid function tests
b) Lower limb X-ray
c) Pelvis X-ray

123. What a 4 years child can do : (Draw square)


a) Draw square 4 years & triangle 5 years
b) Say complete sentence
c) Tie his shoes 5 years

124. What condition is an absolute contraindication of lactation:


a) Mother with open pulmonary TB for 3 month
b) Herpes zoster in T10 dermatome
c) Asymptomatic HIV

125. A child is about to be given FLU vaccine, what allergy should be excluded before giving the vaccine?
a) Chicken
b) Egg
c) Fish
126. Newborn with fracture mid clavicle what is true
a) Most cases cause serious complication
b) Arm sling or figure 8 sling used
c) Most patient heal without complications
 Most clavicles fracture in newborn no need to treatment apart from careful handling.
 If the fracture is displaced and baby in pain, simple sling is require.

127. 5 days old baby vomited dark red blood twice over the past 4 hours. He is active and feeding well
by breast. The most likely cause is:
a) Esophagitis
b) Esophageal varices
c) Gastritis
d) Duodenal ulcer
e) Cracked maternal nipples

128. 8 years old girl presented with fever, numerous bruises over the entire body and pain in both legs.
Physical examination reveals pallor and ecchymosis and petechiae on the face, trunk and extremities.
Findings on complete blood count includes a haemoglobin of 6.3 g/dl, white cell count of 2800/mm 3
and platelet count of 29,000/mm3. Which of the following would be the MOST appropriate diagnostic
test?
a) Hb electrophoresis
b) Bone marrow aspiration.
c) Erythrocyte sedimentation rate.
d) Skeletal survey.
e) Liver and spleen scan.

129. 12 months old baby can do all except:


a) Walk with support one hand
b) Can catch with pincer grasp
c) Can open drawers
d) Response to calling his name
e) Can play simple ball

130. A 5 months old baby presented to ER with sudden abdominal pain and vomiting. The pain lasts for 2-
3 minutes with interval of 10-15 minutes in between. The most likely diagnosis:
a) Intussusceptions
b) Infantile colic
c) Appendicitis

131. 3 years old boy in routine exam for surgical procedure in auscultation discovered low pitch
murmur continues in the right 2nd intercostal space radiate to the right sternal border increased by
sitting & decreased by supine, what you want to do after that?
a) Send him cardiologist
b) Reassurance & till him this is innocent murmur
c) Do ECG
 Innocent Murmur Heart murmurs that occur in the absence of anatomical or
physiological abnormalities of the heart and therefore have no clinical significance.
132. 1 year old baby complaining of acute hepatosplenomegaly, skin bluish nodules and lateral neck
mass. What is the best investigation?
a) liver biopsy
b) Bone marrow aspiration
c) MRI of the chest
d) EBV serology
e) CBC

133. A 6 years old girl presented with low grade fever and arthralgia for 5 days. She had difficulty in
swallowing associated with fever 3 weeks prior to presentation. Physical examination revealed a
heart rate of 150/min and pansystolic murmur at the apex. There was no gallop and liver was 1 cm
below costal margin. The MOST likely diagnosis is:
a) Bacterial endocarditis.
b) Viral myocarditis.
c) Acute rheumatic fever.
d) Pericarditis.
e) Congenital heart failure.

134. 6 days old Neonate not feeding well, lethargic, with urine smell like burned sugar. The diagnosis is:
a) Maple syrup urine syndrome
b) phenylketonurea

135. A 3 years old child woke from sleep with croup, the differential diagnosis should include all except:
a) Pneumonia
b) Tonsillitis
c) Cystic fibrosis
d) Inhaled foreign body

136. Child with positive skin test of TB and previously it was –ve, Treatment of this child?
a) INH alone
b) INH + Rifampicin
c) INH + rifampicin+ streptomycin
d) no treatment

137. 15 years old boy came to your clinic for check-up. He is asymptomatic. His CBC showed: Hb 11.8 g/l,
WBC 6.8 RBC 6.3 (high), MCV 69 (low), MCH (low), and Retic 1.2 (1-3)%, what is the most likely
diagnosis?
a) Iron deficiency anemia
b) Anemia due to chronic illness
c) β-thalssemia trait
d) Sickle cell disease
e) Folic acid deficiency

138. short boy with decreased bon age, most diagnosis is


a) Constitutional delay

139. Mother bring her baby to you when she complain of diaper rash, she went to different drug before
she come to you, she used 3 different corticosteroid drug prescribed by different physician, the rash is
well demarcated & scaly, what is the diagnosis?
a) seborrheic dermatitis
b) contact dermatitis include labi wheras candida not
140. The treatment:
a) Avoid allergen and steroid for contact dermatitis

141. 18 months old child brought to you for delayed speech, he can only say "baba, mama", what's
your first step in evaluating him?
a) Physical examination
b) Delevelopmental assessment.
c) Head CT
d) Hearing test.

142. Baby <2 years age present with a history of URTI, nasal discharge after that complicated to wheezing
& there is rales in the end inspiratory & early expiratory phase ,prolonged expiratory phase , sever
respiratory distress ,using the accessory muscle in respiration, what is the diagnosis:
a) Viral pneumonia.
b) Bronchilitis
c) Bacterial pneumonia

143. Mother bring her baby to you when he present with hematoma in his nail, How to manage
this patient?
a) No need things& ask him to go to the home.
b) Bring a sharp metal & press in the middle to evacuate the Hematoma.
c) Remove the nail

144. Child with morbid obesity, what the best advice for him?
a) Decrease calories intake
b) Dec fat intake
c) Increase fiber
d) Increase water

145. A patient presented with fatigue, loss petite& bloody urine. She gave History of sore throat 3 weeks
back. The most likely diagnosis is:
a) hemorragic pyelonephritis
b) Post streptococcal GN
c) Heamorragic cystitis
d) Membranous GN
e) IgA nephropathy

146. A young girl patient had UTI 1 week ago & received septra (trimethoprime + sulphamethoxazole).
She came with crampy abdominal pain & proximal muscle weakness. The diagnosis is:
a) Polymyositis
b) Gullian parre syndrome
c) Intermittent porphyria
d) Periodic hypokalemic paralysis

147. 2 years old boy with coryza, cough and red eyes with watery discharge (a case of measles). Most
likely diagnosis of the red eyes is:
a) Conjunctivitis
b) Blepharitis
 Cough, coryza, conjunctivitis (red eyes), 40 °C, Koplik's spots seen inside the mouth are
pathognomonic (diagnostic) for measles.
148. Two absolute contraindications to DTP and DTaP:
a) An immediate anaphylactic reaction &encephalopathy within 7 days.
b) Seizure within 3 days of immunization
c) crying within 3 days for 3 or more hours within 48 hours
d) Collapse or shock-like state within 48 hours
e) Temperature ≥ 40.5°C (104°F) within 48 hours
 Contraindications: DTP or DTaP administration
 Absolute:
 Severe reaction following prior DTP or DTaP
 Immediate Anaphylaxis
 Encephalopathy within 7 days of Vaccine
 Relative:
 Moderate Reaction following prior DTP or DTaP
 Fever > 40.5 C within 48 hours of vaccine
 Seizure within 72 hours of vaccine
 Hypotension or Unresponsive Episode within 48 hours
 Inconsolable Crying >3 hours within 48 hours
 Guillain-Barre Syndrome within 6 weeks of vaccine
 Conditions not contraindicating vaccine
 Family History of adverse vaccine event
 Family History of SIDS
 Family of Seizure disorder
 Fever following prior vaccine <40.5 C (105 F)
 If Vaccine Contraindicated, then
 Allergy Testing for anaphylactic reaction
 Administer DT to all other groups

149. Management of obesity in 10 years boy:


a) Multifactorial

150. 2 years baby with gray to green patch in lower back, no redness or hotness, diagnosis is
a) Child abuse
b) No treatment need
c) Bleeding tendency
 Mongolian spot: visible in 6 month and normally disappear to 3-5 years.
 No need treatment.

151. Child normal the doctor discovered by exam that mid sterna murmur at late systolic crescendo-
decrescendo like with wide splitting diagnosis is?
a) Causes include mitral valve prolapse
b) Tricuspid valve prolapse
c) Papillary muscle dysfunction

152. Baby can sit without support, walk by holding furniture. Pincer grasp, pull to stand how old is he
a) 8 months
b) 10 months
c) 12 month
d) 18 month
153. Boy 3 day after flue symptom develop conjunctivitis with occipital and neck L.N enlarged so diagnosis
a) Adenoviral Conjunctivitis
b) Streptococcus
c) HSV

154. The most common cause of failure to thrive in paediatric is


a) Malnutrition

155. Baby complaining increasing haemangioma in the back 2cm:


a) Observation
b) Oral steroid
c) Injection steroid
d) Excision

156. Baby can copy triangle and square what age:


a) 2 years
b) 3 years
c) 5 years

157. Child with dental caries and history of bottle feeding So dd


a) Nurse milk caries

158. Lactating women infected with rubella, management is


a) MMR
b) Stop lactation
 Rubella not protected by postexposure administration of live vaccine

159. 2 months infant with white plaque on tongue and greasy, past history of clamydia conjunctivitis after
birth treated by clindamycin, what is the treatment oral thrush?
a) Oral nystatin
b) Topical steroids
c) Topical acyclovair
d) Oral tetracycline

160. Asystole is one of the non shockable waves what you gonna do is CEAP?
a) CPR
b) Epinephrine
c) Atropine
d) Pacing

161. Group of diseases include, cystic fibrosis, liver failure, the cause is
a) Alpha one antitrypsin deficiency
 α1-antitrypsin deficiency has been associated with a number of diseases:
 Cirrhosis
 COPD, pneumothorax, asthma, emphysema, Bronchiectasis and cystic fibrosis
 Wegener's granulomatosis
 Pancreatitis, Gallstones, Primary sclerosing cholangitis & Autoimmune hepatitis
 Pelvic organ prolapse
 Secondary Membranoproliferative Glomerulonephritis
 Gallbladder cancer, Hepatocellular carcinoma, bladder carcinoma, Lymphoma &Lung cancer
162. Febrile infant with no obvious cause, Do all except:
a) CT scan

163. Child known case of BA moderate intermittent on inhaled salbutamol ,,, about management
a) Add inhaled steroid

164. Child with febrile seizure


a) Give her paracetamol when she had a fever
b) Give her phenobarbiton when she had a fever

165. 2 month old baby on breast feeding Mother asked you about her baby feeding
a) Solid fluid after 4-6 month

166. Child after falling down from bed sustained multiple area of erosion
a) Hemophilia

167. Child present with URTI, lymphadenopathy, splenomegaly ttt :


a) Amoxicillin
b) Supportive treatment only
c) Clindamycin

168. Boy presented to the ER complaining of sudden onset of abdominal pain & leg cramps, he had history
of vomiting 2 days ago, he was dehydrated. Na = 150 , K = 5.4 ,, glucose = 23mmol ,The best initial
investigation is
a) CBC
b) Blood culture
c) ABG ( the Dx is DKA)
d) Urinalysis (dipstick)
e) U/S

169. 3 year old child needs oral surgery & comes to your clinic for check-up. On examination 2/6
continuous murmur, in upper right sternal borders that disappear with sitting , next step:
a) Give AB prophylaxis
b) Ask cardiology consult
c) Clear for surgery
d) Do ECG

170. 17 years old girl missed her second dose of varicella vaccine, the first one about 1 y ago what you'll do
a) Give her double dose vaccine
b) Give her the second dose only
c) See if she has antibody and act accordingly
d) Revaccinate from the start.

171. In a baby with polyhydromnios what could be the cause:


a) Duodenal atresia

172. Child with mild trauma develop hemoarthrosis, in past history of similar episode DX ?
a) Platelets dysfunction
b) Clotting factor deficiency
 Superficial bleeding platelets dysfunction
 Deep bleeding clotting problems
321
173. Infant newly giving cow milk in 9 months old, closed posterior fontanel, open anterior fontanel
with recurrent wheezing and cough, sputum examination reveal hemoptysis, x-ray show lung
infiltration, what is your action?
a) Diet free milk
b) Corticosteroid
c) Antibiotics
 Explanation: infantile pulmonary hemosiderosis.

174. Patient with signs and symptoms of autism what medication to give
 Treatment: A variety of therapies are available, including
 Applied behavior analysis (ABA)
 Medications: Currently, only risperidone is approved to treat children ages 5 - 16 for the irritability and
aggression that can occur with autism. Other medicines that may also be used include SSRIs,
divalproexsodium and other mood stabilizers, and possibly stimulants such as methylphenidate. There
is no medicine that treats the underlying problem of autism.
 Occupational therapy
 Physical therapy
 Speech-language therapy

175. Child with posing head, bowing tibia “rickets”, what is the deficiency?
a) Vitamin D deficiency.

176. 5 years child have congested throat 2 day, complain of painless, clear, vaginal discharge DX>>>>>
a) Foreign body
b) Candida
c) N. gonorrhea
d) Streptococcus (SURE 100%)
e) Tracomanas

177. After delivery start breast feeding :


a) As soon as possible
b) 8hrs
c) 24 hrs.
d) 36 hrs.
e) 48 hrs.

178. A 14 years old boy with type 1 D.M. presented in coma. His blood glucose level is 33 mmol/l. Na is
142 mmol/l, K is 5.5 mmol/l, bicarb is 10 mmol/l. the following are true except : ??
a) The initial Rx. Should be IV normal saline 3l/hourfor1-2hours
b) IV insulinloadingdose1u/kg is necessary.
c) IV Na bicarbonate could be given if pH is 7 or less.
d) Hyprephosphatemia can occur during trement.
e) Hyperchloremia can occur during treatment
 Hyperchloremic metabolic acidosis with a normal anion gap often persists after the resolution of
ketonemia.
 This acidosis has no adverse clinical effects and is gradually corrected over the subsequent 24-48
hours by enhanced renal acid excretion.
 Hyperchloremia can be aggravated by excessive chloride administration during the rehydration phase.
179. 10 years old girl presented with a 2 days history of fever and a 4 cm, warm, tender and fluctuant left
anterior cervical lymph node. The MOST likely diagnosis is
a) Hodgkin’s disease.
b) Acute lymphoblastic leukemia (ALL).
c) Histiocytosis X.
d) Acute bacterial lymphadenitis.
e) Metastatic neuroblastoma.

180. A 7 months old child is brought to your office by his mother. He has an upper respiratory tract
infection for the past 3 days. On examination, there is erythema of the left tympanic membrane with
opacification. There are no other signs or symptoms. What is the MOST likely diagnosis in this patient?
a) Acute otitis media.
b) Otitis media without effusion.
c) Chronic otitis media.
d) Otitis media with effusion.
e) Chronic suppurative otitis media.

181. Nonbilious vomiting that increase in volume and frequency is seen


a) Alkalosis >> low K+, low chloride and metabolic alkalosis.
 Unconjugated hyperbilirubinemia is also present.

182. Which of the following medications has been shown to be safe and effective for migraine prophylaxis
in children?
a) Propranolol
b) Fluoxetine.
c) Lithium.
d) Naproxyn.
e) Timed-released dihydroergotamine mesylate (DHE-45).

183. Term baby born to a mother who developed chickenpox 7 days before delivery. The baby is
a symptomatic, which is true?
a) Give acyclovir 15 mg /kg I.V Q 8 hr. for 7 days immediately
b) Give acyclovir & varicella zoster immune globulin when the baby develops symptoms.
c) Serologic evidence is needed before initiation of therapy
d) The mother & baby should be nursed together at their own room
e) None of the above.
 15% of pregnant women are susceptible to varicella (chickenpox). Usually, the fetus is not affected,
but is at high risk if the mother develops chickenpox:
 In the 1st half of pregnancy ( < 20 weeks ), when there is a < 2 % risk of the fetus developing
sever scarring of the skin & possibly ocular & neurological damage
 Within 5 days before or 2 days after delivery, when the fetus is unprotected by maternal antibodies &
the viral dose is high. About 25 % develop a vesicular rash. Exposed susceptible women can be
protected with varicella zoster immune globulin & treated with acyclovir. Infants born in the high-risk
period should also receive zoster immune globulin & are often also given acyclovir prophylactically.

184. 4 years old girl, decrease head growth, decrease social intraction, decrease in language …etc:
a) Rett's syndrome

322
185. A 48 hours old newborn infant in critical care unit with respiratory distress & Jaundice. Hb 9g/dl,
retic 4%. Maternal Hx of previous normal term pregnancy without transfusion, Blood typing shows
hetero specificity between mother and child. Indirect Coomb’s test is +ve. The most probable
Dx is:
a) Thalassemia
b) Maternal-Fetal blood group incompatibility
c) Sickle cell anemia
d) Septicemia
e) Hereditary Red cell enzyme defect.

186. A 6 years old girl is brought to the family health center by her mother. The child today had sudden
onset of a painful sore throat, difficulty swallowing, headache and abdominal pain. The child has had no
recent cough or coryza and was exposed to someone at school that recently was diagnosed with a
“strep throat”. On examination the child has a temperature of 40oC. She has tender anterior
cervical nodes and exudative tonsils. The lungs, heart, and abdominal examination are benign. What
treatment would you offer for this child?
a) Zithromax
b) Penicillin V
c) Ciprofloxacin
d) No antibiotics, rest, fluid, acetaminophen, and saline gargles.
e) Trimothoprim.
 In URTI there’s a McIsaac criterion (weather or not to start antibiotics): no cough, tender
anterior cervical L.N., erythmatous tonsils with exudates, fever> 38, age 3-14. if 0-1 no culture no
antibiotics, 2-3 culture if positive antibiotics, 4 start antibiotics. And in this cause 4 are present..
 Treatment is by penicillin V if allergic erythrmomycin.

187. Development in children, all are true EXCEPT:


a) At 1 year can feed himself by spoon)

188. Which one of the following component causes contact dermatitis in children?
a) Citric acid
b) Cinnamon
 Primary Contact Dermatitis: is a direct response of the skin to an irritant. The most common irritants
are soap, bubble bath (may cause severe vaginal prorates in prepubertal girls), saliva, urine, feces,
perspiration, citrusjuice, chemicals (creosote, acids) &wool.
 Allergic Contact Dermatitis: requires reexposure of the allergen and characterized by delayed
hypersensitivity reaction.
 The most common allergen implicated include poison ivy, poison oak &poison sumac (rhus dermatitis),
jewelry (nickel), cosmetics (causing eye lid involvement) &nail polish, topical medications [neomycin,
thimerosal, calamine, para-aminobenzoic acid (PABA)], shoe material (rubber, tanning agents, dye)
and clothing materials (elastic or latex compounds).

189. To prevent tetanus in neonate:


a) Give anti-tetanus serum to neonate
b) Give immunoglobulin to mother
c) Give tetanus toxoid
d) Give antibiotics to mother
e) Give penicillin to child to kill tetanus bacilli
 DTP= diphtheria, tetanus & pertussis D&T are toxoids, P is inactivated bacteria Route
190. A 6 year old girl developed day time wetting for 2 days. She is fully toilet trained. She is afebrile & dry
for 4 years. The most appropriate diagnostic measure is:
a) Bladder US
b) Examination of vaginal vault
c) Urine analysis & culture
d) Urine specific gravity
e) Voiding cysto-urethrography
 Lab Investigations:
 Urinalysis and the specific gravity of urine should be obtained after an overnight fast and evaluated to
exclude polyuria secondary to diabetes as a cause of frequency and incontinence and to determine if
there is normal concentrating ability.
 Urine culture will determine the presence or absence of a urinary tract infection, which, when treated
could improve continence.
 If daytime wetting is occurring, a renal and bladder ultrasound may help rule out possible outlet
obstruction
 Spine imaging or MRI may determine if there is a neurological cause.

191. 7 months old boy presented with history of interrupted feeds associated with difficulty in breathing
and sweating for the last 4 months. Physical examination revealed normal peripheral pulses,
hyperactive precordium, normal S1, loud S2 and Pansystolic murmur grade 3/6 with maximum
intensity at the 3rdleft intercostal space parasternally. The MOST likely diagnosis is:
a) Small PDA (Patent ductus arteriosus).
b) Large ASD (Atrial septal defect).
c) Aortic regurgitation
d) Mitral regurgitation.
e) Large VSD (Ventricular septal defect).

192. All of the following are true about pyloric stenosis, EXCEPT:
a) Incidence male more than female
b) Onset is generally late in the first month of life
c) Vomitus is bile stained
d) Appetite is good
e) Jaundice occur in association

193. Composition of standard and reduced osmolarity ORS solutions, The amount of Na+ in ORS
“oral rehydration solution” in (WHO) is:
a) 150 meq
b) 120
c) 90
d) 60
e) 30
194. Risk factor of sudden death syndrome includes all of the following, EXCEPT:
a) Cigarette smoking during pregnancy
b) Old primigravida
c) Crowded living room
d) Prematurity
e) Small gestational age
 Potential risk factors include
 smoking, drinking, or drug use during pregnancy
 poor prenatal care
 Prematurity or low birth-weight
 Mothers younger than 20
 Smoke exposure following birth
 Overheating from excessive sleepwear and bedding
 Stomach sleeping

195. Symptoms of cystic fibrosis in neonate:


a) Meconium ileus
b) Pneumothorax
c) Steatorrhea
d) Rectal prolapse
 Meconium ileus is associated with CF (defect in chromosome 7, autosomal recessive)

196. DKA in children, all of the following are true EXCEPT:


a) Don’t give K+ till lab results come
b) ECG monitoring is essential
c) If pH < 7.0give HCO3-
d) NGT for semiconscious pt
e) Furosemide for patient with oligouria
 Give fluid (volume resuscitation) is the goal. Polyuria is one of DKA symptoms, not oliguria.

197. Hypothyroid in young baby usually due to:


a) Endocrine irresponse
b) Enzyme deficiency
c) Drug by mother
d) Agenesis
 Missing or misplaced thyroid gland
 Most babies with CH are missing their thyroid gland or have a thyroid that did not develop
properly. In some cases, the thyroid gland may be smaller than usual or may not be located in the
correct place.
 In healthy people, the thyroid gland is located in the center of the front of the neck, near the top of
the windpipe. In some children with CH, the thyroid gland may instead be under the tongue or on the
side of the neck. If the thyroid gland is in the wrong place, or if it is underdeveloped, it often does not
work well and makes less thyroid hormone than needed by the body.
 If the thyroid gland is missing, the baby cannot make any of its own thyroid hormone. A missing,
underdeveloped or misplaced thyroid gland is a birth defect that happens for unknown reasons
and is usually not inherited.
198. MMR given at age of:
a) 3 months
b) 8 months
c) 12 months
d) 24 months

199. 6 months old patient with sepsis, the most likely organism will be:
a) Listeria.
b) Hemolytic Streptococci.
c) H. Influenza type B.
d) Staph. Epidermis.

200. Hospitalized child (on chemotherapy) and when start IV access develop sepsis organism:
a) E. coli
b) Pseudomonas
c) strep

201. All are vaccines given in Saudi Arabia to normal children EXCEPT:
a) TB.
b) Pertussis.
c) H. Influenza type B (HiB).
d) Mumps.
e) Diphtheria

202. Neonatal just delivered, term pregnancy. Developed respiratory distress CXR showed
multicystic lesion in Lt side shifted mediastinum to the Rt , decreased bilatral breath sound & flat
abdomen:
a) Diaphragmatic hernia
b)RDS
c) Emphysema

203. Child with BMI 24.4


a) Normal BMI

204. 2 months boy with projectile vomiting. On examination olive mass in right upper quaderant
of abdomen. 1st step of investigation is:
a) X-ray abd.
b) U&E
c) Barium study
d) US

205. Sign of congestive heart failure in children all .EXCEPT


a) Gallop rhythm
b) Periorbital edema
c) Basal crept.
d) Hepatomegaly
e) Bounding pulse

206. Treatment of meningitis:


a) Amoxycilin
b) Deoxycillin
c) Ampicillin
207. Treatment of tetralogy of fallot ,all true EXCEPT
a) Thoracotomy
b) Use of systemic antibiotics.
c) Chest tube insertion.
 Definitive management is total correction of pulmonary stenosis and VSD this can be performed
even in infancy
 Blalock shunt if pulmonary arteries are excessively small, to increase pulmonary blood flow
and decrease hypoxia
 This consists by creation of shunt from a systemic to pulmonary Artery by anastomosis
between subclavian to pulmonary artery(pulse is not palpable on ipsilateral side after
procedure)
 Antibiotic prophylaxis for endocarditis
 Fallot's spells need propranolol
 Vasodilators should be avoided.

208. Child presented with history of restless sleep during night, somnolence "sleepiness" during day
time, headache etc the most likely diagnosis is
a) Sinopulmonary syndrome
b) Sleep apnea
c) Laryngeomalacia
d) Adenoidectomy.
 Tonsillitis and enlarged adenoids may occlude the nasopharyngeal airway especially during sleep, this
results in obstructive sleep apnea, the child will present with loud snoring punctuated by periods of
silence followed by a large gasp and as a complication of interrupted sleep ,child will have somnolence
and sleep during the day
 Laryngeomalacia: the stridor starts at or shortly after birth and is due to inward collapse of soft
laryngeal tissue on inspiration. It usually resolves by the age of 2 or 3yrs, but meanwhile the baby may
have real respiratory difficulties, diagnosis is confirmed by laryngoscopy.

209. Child attended the clinic 3 times with history of cough for 5 days, he didn't respond to symptomatic
treatment, which of the following is true in management?
a) CXR is mandatory
b) Trial of bronchodilator
c) Trial of antibiotics
 Cough is the most common symptom of respiratory disease and indicates irritation of nerve receptors
in pharynx, larynx, trachea or large bronchi. While recurrent cough may simply indicate that the child
is having respiratory infection, in addition to other causes that need to be considered

210. Meningitis in childhood, all are true, EXCEPT:


a) Group B streptococci and E.coli are the most common cause in neonates.
b) H. influenza meningitis can be treated by ampicillin or chloramphenicol.
c) Present with specific signs in neonates
d) If pneumococcal meningitis, Rifampicin is given to contact.
 May be (b)if we consider that H.influenza is becoming resistant to penicillin, but if we consider that
it is an old question ,then, it is true information and the answer will be (d).
 The most common pathogens in neonates are: E.coli, group B streptococci and L.monocytogenous.
 Chemoprophylaxis of contacts is not necessary to prevent the spread of pneumococcal meningitis.
However, chemoprophylaxis is an important aspect of prevention of invasive pneumococcal
infections in children with functional or anatomic asplenia (e.g. SCD). Besides, the prophylaxis will be
with penicillin not with rifampin
211. 2 weeks old infant with jaundice, cirrhosis and ascites, the cause is:
a) Gilberts disease
b) Criggler-najjar syndrome
c) Congenital biliary atresia
d) Dubin
212. Whooping cough in children, all are true EXCEPT:
a) Absolute lymphocytosis.
b) Can cause bronchiectasis.
c) Patient is infective for 5 weeks after onset of symptoms.
 The incubation period is typically seven to ten days in infants or young children, after which there are
usually mild respiratory symptoms, mild coughing, sneezing, or runny nose. This is known as
the catarrhal stage. After one to two weeks, the coughing classically develops into uncontrollable fits,
each with five to ten forceful coughs, followed by a high-pitched "whoop" sound in younger children,
or a gasping sound in older children, as the patient struggles to breathe in afterwards
(paroxysmal stage).
 Persons with pertussis are infectious from the beginning of the catarrhal stage (runny nose, sneezing,
low-grade fever, symptoms of the common cold) through the third week after the onset of
paroxysms (multiple, rapid coughs) or until 5 days after the start of effective antimicrobial treatment.
 Common complications of the disease include pneumonia, encephalopathy, earache, or seizures
 Most healthy older children and adults will have a full recovery from pertussis; however those
with comorbid conditions can have a higher risk of morbidity and mortality.
 Infection in newborns is particularly severe. Pertussis is fatal in an estimated 1.6% of hospitalized
infants who are under one year of age. Infants under one are also more likely to develop
complications (e.g., pneumonia (20%), encephalopathy, seizures (1%), failure to thrive, and death
(0.2%). Pertussis can cause severe paroxysm-induced cerebral hypoxia and apnea. Reported
fatalities from pertussis in infants have increased substantially over the past 20 years
213. Child brought by his father looks pale doesn’t like to meat. Hypochromic microcytic anemia
a) Bone Marrow biopsy
b) Transferi
c) Daily iron and vitamin
214. About Kernicterus, all are true EXCEPT:
a) Can occur even if neonate is 10 days old.
b) It causes neurologicl abnormalities, it can be reversed.
c) Can cause deafness.
d) All types of jaundice cause it.
 Kernicterus: Severe hyperbilirubinemia TSB>25-30 mg/dl (428-513 micromol/l) is associated
with increased risk of Bilirubin-Induced Neurological Dysfunction ( BIND) which occurs when
bilirubin crosses BBB & bind to brain tissue.
 The term acute bilirubin encephalopathy (ABE) is used to describe acute manifestation of BIND,
the term '' KERNICTERUS'' is used to describe the chronic & permanent sequelea of BIND.
So, rgarding the choice (b) is not a rule b/c early detection can prevent permanent
neurological deficit & reverse the acute (ABE) but the ''KERNICTERUS'' is a term used to
describe the chronic sequelea.
 Age: Acute bilirubin toxicity appears to occur in the 1st few days of life of the term infant.
Preterm infants may be at risk of toxicity for slightly longer than a few days. If injury has occured,
the 1st phase of acute bilirubin enchephalopathy appears within the 1st week of life.
 Complications of kernicterus: Extrapyramidal system abnormalities, auditory dysfunction, gaze
dysfunction, dental dysplasia.
215. Infant brought by the mother that noticed that the baby has decreasing feeding, activity and
lethargic on examination febrile (39), tachycardia, his bp 75/30, with skin rash. DX:
a) Septic shock

216. 4 years old child what can he do :


a) Copy square and triangle
b) Speak in sentences

217. Regarding child with moderately severe asthma, all are true EXCEPT:
a) PO2<60
b) PO2>60
c) Low Bicarb. Level
d) IV cortisone can help.

 Moderately-severe asthma: The R.R. is increased. Typically, accessory muscles of respiration are used,
and suprasternal retractions are present. The H.R. is 100-120 b/min. Loud expiratory wheezing can
be heard.
 Pulsus paradoxes may be present (10-20 mm Hg). Oxyhemoglobin saturation with room air is 91-95%.
250 cases in clinical medicine:
 Indicators of VERY SEVERE, LIFE-THREATENING attack (NOT moderately – severe attack):
 Normal (5-6 kPa, 36-45 mmHg) or increased CO2 tension.
 Severe hypoxia of LESS than 8 kPa (60 mmHg).
 Low pH.
 In very severe, life threatening attack: Normal or increased PCO2-----Low pH (resp. acidosis) --High
Bicarb, level
 In moderately severe attack:
Hyperventilation low PCO2 -High pH (resp. alkalosis) --Low Bicarb. Level.

218. Child, ingested acoustic material, looks ill and drooling what is your immediate action:
a) Antibiotics
b) Endoscopy
c) Chelating agent
d) Airway assessment

219. A blood transfusion given to child who then developed a bleed, what is the cause:
a) ↓prothrombin
b) ↑fibrinolytic activity
c) ↓ca++
d) ↓Fibrinogen
e) ↓platelets

 Bleeding due to depletion of platelets and clotting factors in stored blood


 Fibrinogen deplete faster than platelets →answer is ↓fibrinogen Treatment first is FFP if
not corrected then platelet transfusion

220. 3 years old his parents has TB as a pediatrician you did PPD test after 72 hr you find a 10mm
enduration in the child this suggest
a) Inconclusive result
b) Weak positive result
c) Strong positive result
221. A child came to ER due to haematuria after history post strept GN, so the diagnosis test :
a) LowC3
b) Increase BUN creatinine
c) Streptozyme
 Diagnosis depends on:
 +ve pharyngeal or skin culture
 ↓complement.
 Elevated BUN and creatinine values
 ASO titers are frequently used to document streptococcal infection, but a more sensitive test is the
streptozyme test, which tests antibodies to ASO, anti–DNAse B, AHase, and anti-NAD.

222. Meningitis, CSF : Glucose normal, protein high, high leukocytes mainly lymphocytes 70:
a) Viral meningitis

223. Risk factor for HSV II accusation in infants all of the following EXCEPT:
a) Cervical transmission is commoner than labial
b) Maternal first episonde is of greater risk than recurrence
c) Maternal antibodies against HSV I protect from HSV II
d) Head electrodes increase risk of infection
 Neonatal herpes simplex encephalitis the predominant pathogen is HSV-2 (75% of cases), which is
usually acquired by maternal shedding (frequently asymptomatic) during delivery. A preexisting but
recurrent maternal genital herpes infection results in 8% risk of symptomatic infection, usually
transmitted at the second stage of labor via direct contact. Should the mother acquire genital herpes
during pregnancy, the risk increases to 40%.
 The absence of a maternal history of prior genital herpes does not exclude risk; in 80% of cases of
neonatal HSE, no maternal history of prior HSV infection is present. Prolonged rupture of the
membranes (>6 h) and intrauterine monitoring (eg, attachment of scalp electrodes) are risk factors.
 In about 10% of cases, HSV (often type 1) is acquired post-partum by contact with an individual who is
shedding HSV from a fever blister, finger infection, or other cutaneous lesion

224. 5 years old child with abdominal pain after 2 wks of URTI, HB 8, retics 12% WBC NL peripheral blood
smear showed target cells, RBC inclusions dx:
a) SCA (the only hemolytic anemia in the answers)
 This child has a vaso occlusic crysis of SCA that caused by URTI, Hiehg retic>> hemolytic, Target>> SCA
Iinclusion>> fuctional aspleensim ( which is occure in SCA)

225. Most common cause of failure to thrive?


a) Asthma
b) Intolerance to failure to proteins and carbohydrates
c) Cystic fibrosis
d) Low socioeconomic status
e) GERD

226. 8 month child with coryza, fever, cough, T 38 c. best management is:
a) Paracetamol + culture sensitivity
b) Admission and start parenteral Antibiotic

331
227. child developed fever, headache after rupture of maculi lesion on his face
a) Varicella
b) HSV1
c) HSV2
d) CMV
e) Rubella

228. 12 years old boy with jaundice &increase indirect bili dx :


a) Autoimmune hepatitis
b) Glibert

229. DPT vaccine shouldn’t give if the child has:


a) Coryza
b) Diarrhea
c) Unusual cry
d) Fever = 38

230. Vasoconstrictive nasal drops complication :


a) Rebound phenomenon

231. Epididymitis:
a) Common at the age 12-18
b) Iliac fossa pain
c) Scrotal content does not increase in size.
d) Ultrasound will confirm the diagnosis.
e) None of the above

232. Childhood asthmaall are true except:


a) 90% bronchospasm are induced by exercise.
b) Inhalation of beclamethasone is safe.
c) Inhalation by aerospace champer in younger child.
d) Hypercapnia is the first physiological change.
e) Cough is the only symptom.
 Regarding A: Upper respiratory tract infection is the most common cause of asthma exacerbations!!!
 Not broncohospasm only which is not a complicated problem!
 B, C and D are correct
 Cough (nocturnal usually) can be the only symptom but cyanosis, SOB, wheezing. . .etc. can occur.

233. Diarrhoea can occur in all the following, EXCEPT:-


a) Hypothyroidism
b) Hyperthyroidism

234. 18 months baby can typically do the following except:


a) Have a vocabulary of 10 words
b) Build a ten brick tower.
c) Drink from a cup.
d) Feeds himself with a spoon.
 Explanation: Can build a tower of 2 - 3 blocks, can use a spoon & cup and can say 10 words.
235. Acute gait disturbance in children, all of the following are true EXCEPT:
a) Commonly self-limiting
b) Usually the presenting complaint is limping
c) Radiological investigation can reveal the Dx
d) Most often there is no cause can be found

236. All the following can cause small stature in children except:
a) Hypothyroidism
b) Tunner syndrome
c) Klinefenter syndrome
d) Down syndrome

237. In new born ,the following needs immediate treatment:


a) Asymptomatic Hydrocele
b) Erupted tooth
c) Absent femoral pulse

238. A 6 weeks old infant presented with yellowish eye discharge and persistant tearing of one eye since
birth, all of the following are true Except
a) Treatment include sulphacetamide ointment daily
b) Advice the mother to do warm massage
c) Can be treated by systemic antibiotics
d) Do probing to bypass the obstruction

239. Mother came with her child who had botulism, what you will advise her
a) Never eat canned food again
b) Store canned food at home
c) Boil canned food for 40-50 min
d) Check expiry date of canned food

240. Which of the following is describe the normal developmental stage for 6 months old child :
a) Sits without support
b) Rolls front to back
c) No head lag.(3 months)
d) Stand alone. (1 year)

241. Case about diagnosis of acute lymphocytic leukaemia ALL


 The total number of white blood cells may be decreased, normal, or increased, but the number of
red blood cells and platelets is almost always decreased. In addition, very immature white blood
cells (blasts) are present in blood samples examined under a microscope.
 A bone marrow biopsy almost always done to confirm the diagnosis and to distinguish ALL from
other types of leukemia

242. Child fell on her elbow and had abrasion, now swelling is more, tenderness, redness, swelling
is demarcated (they gave dimensions) child has fever. Dx:
a) Gonoccal arthritis
b) Synovitis
c) Cellulitis of elbow

243. Baby with vesicles on the face and honey comb crust which of the following organism cause it:
a) Staph aurous
332
244. You are supposed to keep a child NPO he's 25 kgs, how much you will give:
a) 1300
b) 1400
c) 1500
d) 1600

245. Parents brought their baby to you who is on bottle feeding. On exam whitish lesion on either side of
teeth seen with blackish lesion on maxillary incisors and second molar teeth. There is history of
leaving the baby with bottle in his mouth during sleeping. The Dx:
a) Nursery dental caries
b) Gingvostomatis

246. Most common tumor in children


a) ALL
b) Rabdomyosarcoma
c) Wilm's tumor
 ALL : most common childhood tumor
 Rabdomyosarcoma : most common soft tissue tumor
 Wilm's tumor: most common intra- abdominal childhood tumor

247. Child with moderate asthma and he on b2agnosit what you will add to decrease the recurence of
asthma attacks
a) Corticosteroids inhaler

248. 6 years old boy, eat the paper and soil, best initial ttt is:
a) Fluxitin.
b) Behavioral therapy.

249. Neonate with mucopurulant eye discharge lid swelling and culture positive for gm –ve diplococcic
, treatment
a) intravenous cephalosporin

250. Child came to ER with fever, stridor, x-ray showed swollen epiglottis, in addition to oxygen, what u
will do?
a) Throat examination.
b) An emergency tracheostomy.
c) Endotracheal intubation
d) Nasopharyngeal intubation.

251. Child with aspirin intake overdose ...what kind of acid base balance:
a) Metabolic alkalosis wt respiratory
b) Metabolic acidosis wt respiratory alkalosis
c) Respiratory alkalosis with metabolic acidosid
d) Respiratory acidosis with metabolic alkalosis

252. Eight years old child with late systolic murmur best heard over the sterna border, high
pitch, crescendo, decrescendo, diagnosis is
a) Physiological murmur
b) Innocent murmur
c) Ejection systolic murmur
d) Systolic regurgitation murmur
253. Child was presented by congested throat , coryza , high grade fever , which of the following is
true regarding this condition :
a) Viral > bacterial
b) Bacterial > viral
c) Antibiotics should be given any way
d) It is most likely due to EBV

254. Baby with red macule & dilated capillary on the right side of the face
a) Sturge-Weber Syndrome or Nevus Flammeus
b) Milia or cavernous haemangioma

255. 3 years old with symptoms of acute urinary tract infection which of the following you would like to
do in this acute state:
a) Renal U/S
b) Folly catheter
c) VSUG

256. Child with fever and runny nose, conjunctivitis and cough then he developed Maculopapular rash
started in his face and descend to involve the rest of the body:
a) EBV
b) Cocxaci virus
c) Rubella virus
d) Vaccini virus

257. 4 years old child, was diagnosed as SCD, so many times came to hospitals with, dyspnoea,
dactylites, (he put sign of acute crises ), the best strategy for prolonged therapy is:
a) IV hydration fluids with analgesia.
b) Follow in Out pt clinic
c) Refer to tertiary haem center.

258. Children while he was playing a football , the ball hit his hand from lateral fingers, after a while
the child complains pain and swelling on those fingers and painful middle finger with hyperextension
of interphalengial joint, swelling was more in the DIP and IP Joints , also , there was pain on his palm,
what is the most likely cause:
a) Rupture of profound muscles in hand
b) Rupture of superficial muscle

259. child transfer to another city and attend a new school he lose his attention and doesn’t
react with colleagues the most appropriate description:
a) Adjustment syndrome

260. Child has 39 fever, red tonsils with no exudate , slightly enlarged LN but not tender
a) Could be viral or bacterial
b) It is unclear so start antibiotic
c) It is more likely viral
d) It is more likely bacterial
e) It is EBV

261. Neonate with bilious vomiting, don’t pass faces next investigation:
a) Barium enema
b) PR examination ( R/O Hirshprung dis. )
262. Child in well-baby clinic can name 4 colour say 5 word, hop on one leg .what is the age?
a) 48 months

263. Child was playing and felt in the toy, his leg rapped and twisted he don’t want to
walk since yesterday?
a) Ankle tissue swelling
b) Spiral tibial fracture
c) Chip tibial fracture
d) Femur neck of the tibia fracture

264. 4 years old child loss his skill and became isolated
a) Autism
b) Aspirgl

265. To prevent infection in new born


a) Hand wash between examine every child
b) Wear Gloves
c) Overshoes
d) Culter from equipment

266. most common parotid gland tumour in children is


a) Mixed tumor (pleomorphic adenoma)

267. what is the most common malignant parotid tumour in children:


a) Mucoepiptheloild carcinoma
b) Adenocarcinoma
c) Undifferentiated CA
d) Undifferentiated sarcoma

268. Child 3 years old fell from the bed vomited twice and has mild headache and no loss
of consiousness..you will:
a) call for neurologist
b) Send home with close observation
c) CT scan
d) MRI

269. 2 years old child with hair loss in the temporal area and boggy swelling “ I think was 3 cm !! ,
multiple pustules … ?
a) Trichotillomania
b) Aplasia cutis congenital
c) Kerion
d) Favus

270. Patient talking to doctor and the pt look to his right side most of the time, when the doctor asked him
why is that? He said that his mother is there but in fact no one is there, after asking the pt family they
said that the mother died when he is child Dx?
a) Visual hallucination
b) Auditory hallucination
c) Psychosis
271. Child on nutritional supplementation came to ER with 2 hours, hx of vomiting, nausea, abd. Pain,
diarrhea, vomitus was black in color. Dx:
a) Hypervitaminosis
b) Iron overdose

272. Child after his father died start to talk to himself , walk in the street naked when the family asked him
he said that his father asked him to do that , he suffer from those things 3 days after that he is now
completely normal and he do not remember much about what he did Dx ???
a) Schizophrenia
b) Schizoaffective
c) Psychosis
d) There was a fifth choice I do not remember it, I think they make from his father death a cause.
 Explanation: It could be transient hallucination under stress, See oxford handbook of psychiatry 3 rd
edition page 658 .

273. Child with high-grade fever for 5 days and sore throat, on examination there was tonsillitis and
white patches on the gingiva. No LN enlargement, ASO is negative. The most likely causative organism
is:
a) Coxsackie virus.
b) Herpes simplex virus.
c) EBV.

274. A child presented with Mild gastroenteritis, you will manage him with:
a) 50 mL/kg of ORS over first 4 hours, then 50 ml with each bowel motion or as a maintenance .
b) 50 mL/kg of ORS over first 4 hours, then 100 ml with each bowel motion.
c) 100 mL/kg of ORS over first 4 hours, then 50 ml with each bowel motion.
d) 100 mL/kg of ORS over first 4 hours, then 100 ml with each bowel motion.
 Maintenance fluid is different and it depends on the age of the patient (500 mL/day for children
younger than 2 years, 1000 mL/day for children aged 2-10 years, and 2000 mL/day for children older
than 10 years) I think they mentioned the age of the child but I can’t recall it. Also, in the
choices I'm not sure if it says (with each bowel motion or as a maintenance)

275. Child presented with diarrhea, fatigue, abdominal pain and jaundice for few days. There was
history of drinking contaminated water. The most likely organism is:
a) Hepatitis A.
b) Hepatitis C

276. Infant come after 5 weeks with difficult breathing and occasionally turns to blue. On examination
there was pansystolic murmur, most likely Dx:
a) Large VSD
b) ASD
c) Left ventricle hypoplastic.

277. A child presented with sore throat and fever. She had history of impetigo and was resolved
completely during an appropriate course of antibiotic, ASO titer was positive. The same antibiotic was
prescribed to her condition, and the proper duration for such case is:
a) 5 days.
b) 7 days.
c) 10 days. (Answered by Pediatric ID consultant)
278. Child with flu like illness with red non-blanching papules :
a) ITP
b) Infectious mononucleosis

279. A mother came to your clinic and she worries about her overweight child. Your advice is that the
best way helps in losing weight is to:
a) Decrease calorie intake.
b) Increase water intake.
c) Eat a lot of vegetables

280. 3 months Child with low-grade fever, wheezing. CXR shows hyperinflation and some infiltrate what’s
the diagnosis?
a) CROUP
b) Epiglottitis
c) Bronchial asthma

281. child with low grade fever , sore throat in examination there is lymph node enlargement but
not tender and no exudate on pharynx DX
a) It is most likely streptococcal than viral
b) It is viral more than bacterial
c) Most likely EBV

282. Pediatric patient from developing country presented with muscle wasting, weight loss and absent
edema. What is the diagnosis:
a) Marasmus
b) Kwashiorkor
c) Muscle wasting syndrome
Marasmus kwashiorkor
present of muscle wasting presence of edema
Body weight less than 80%of average increase in >18month
weight. muscle wasting syndrome:
absence of edema Loss of weight.
increase prior to age 1 Muscle atrophy.
In older pt with chronic disease.

283. Infant with high grade fever, Irritable, Look sick.. Complain of anuria 4 hour with multiple
petechiea and purpura on body. He was tachycardic and hypotensive DX
a) Renal failure
b) Septic shock

284. 5 years old baby presented with his parents with pallor his HB is 9, he has microcytic
hypochromic anemia, no other complain. What you will do for him?
a) iron therapy and close observation
b) Daily multivitamins with iron

285. 3 months old baby brought by his parents complaining of abdominal distention bilious vomiting,
constipation, the parents informed that the constipation has been an issue since his birth, what is the
single diagnostic investigation to do ?
a) Barium enema
b) Plain x-ray
c) Rectal examination
286. Child with inferior and pain but with normal movement of knee, no effusion on knee what the
important thing to do?
a) Blood culture
b) ESR
c) ASO titer
d) Aspirate from knee joint
e) Plain film on thigh

287. Perthes disease all except


a) Can be presented with painless limp
b) It always unilateral
 Legg-Calve-Perthes disease (LCPD) is avascular necrosis of the proximal femoral head resulting from
compromise of the tenuous blood supply to this area. LCPD usually occurs in children aged 4-10
years. The disease has an insidious onset and may occur after an injury to the hip. In the vast
majority of instances, the disorder is unilateral. Both hips are involved in less than 10% of cases,
and the joints are involved successively, not simultaneously.

288. Child complaining of fever, sore throat all examination was normal What is the treatment?
a) Cefuroxime
b) Ceftriaxone
c) Give paracetamol and take pharynx swab

289. IV fluid (LR) candida be given at age:


a) 3 months
b) 8 months
c) 12 months
d) 24 months

290. A baby with blood in the stool and bought of crying and x ray shows obstructive pattern.. looks like
intussusception you will do:
a) Surgery
b) Barium enema
c) Observation
d) Give IV fluids and let obstruction solve itself

291. The most common cause of epistaxis in children is:


a) Nasal polyps
b) Self-induced
292. Newborn Apgar score 3 (cyanotic, limp, decrease breathing, HR less than 60) your action:
a) Volume expansion
b) Chest expansion
c) Ventilation
d) Bicarbonate
e) Warm & dry

293. One of the following manifests as croup:


a) Foreign body
b) Pneumonia
c) Common cold
d) Asthma
294. Malaria in a child:
a) Crescent shape gametocyte of vivex is diagnostic in the stool
b) The immediate treatment primquine for 3 days
c) 72h treatment of malaria is suffeceint
d) The most common cause is falciparum

295. Scaly purple lesions in the face of a child the cause


a) Staphylococcus aurous
b) Beta hemolytic streptococcus
c) H. influenza

296. Regarding group A streptococcus. Infection have lead to rheumatic fever :


a) Blood dissemination
b) By causing pharyngitis, tonsillitis.
c) Joint invasion.
d) Affect skin.
e) Reach endocardium

297. Treatment to increase fetal hemoglobin in sickle cell disease :


a) Hydroxyurea

298. Child with leukemia he has septicemia from the venous line the organism is:
a) E coli
b) GBS
c) Pseudomonas

299. 6 years old child presents with straddling gait and in ability to stand or walk without support, he is
irritable with vomiting 3 times; he has a history of chickenpox 3 weeks ago. O/E all are normal
except resistance when trying to flex the neck, what is the most likely diagnosis:
a) Fradrich's ataxia
b) Acute cerebellar ataxia
c) Meningioecephalitis
d) Gillian Barre syndrome

300. Child with garlic smell:


a) Alcohol toxicity
b) Organophosphate toxicity
c) Caynide toxicity

301. Newborn presented with conjunctivitis and O.M , what’s the treatment?
 I guess this is a case of infection with chlamydia intrauterine , they asked about several AB
there is no doxycycline nor erythromycin

302. Newborn has vomiting after every meal intake. The examination was normal and the only
abnormality was dehydration. No other clinical signs. No tests ordered yet. What will you do?
a) Order abdominal CT
b) Reassure the parents
c) Refer to GS
d) Discharge on ORS
303. 3 months baby with history of bronchiolitis, what is the cause?
a) RSV

304. Child with SOB and runny nose came with fever (38 ) all the sign of respiratory distress there ..
There is diffuse wheezing on the chest with prolonged expiration and inspiratory cracles ,, diagnosis:
a) Viral pneumonia
b) Bronchiolitis
c) Croup
d) Bacterial pneumonia

305. 5 year old child had abdominal blunt trauma, doctor confirm presence of internal hematoma in 1st
and 2nd part of duodenum, high amylase mx?
a) CT-guided hematoma drainage
b) Duodenal resection
c) Exploratory laparoscopy
d) Don’t remember. I think conservative

306. 5 months old baby , in ER with sudden abdominal pain , pain last 2-3 min with intervals of 10-15
mins between each attack
a) Intussesption
b) Infantile colic
c) Appendicitis

307. Child with small macules mainly in the chest plus knee and elbow arthritis, Diagnosis?
a) Juvenile RA
b) Infectious arthritis

308. Child presented with jaundice, vomiting, hepatomegaly.. etc. What hepatitis virus is more likely to be
the cause:
a) A
b) B
c) C

309. Child with iron toxicity, best way of management:


a) Gastric lavage
b) Ipecac syrup
c) Magnesium citra
 Forgot the rest of choices, but there was no deferroxamine or charcoal

310. What vitamin you will give to prevent hemorrhagic disease of newborn?
a) Vitamin k

311. Rash description: Dew drops on rose petal?


a) Measles
b) Rubella
c) Urticaria
d) Chicken pox
e) Psoriasis

341
312. 1 month child with vomiting, abdominal distension, and constipation since birth, next step
in diagnosis:
a) Digital rectal examination
b) US

313. Child with non-bilious vomiting and abdominal distension. On exam. Small mass in epigastric area. X-
ray shows double bubble:
a) Pyloric stenosis

314. In child sleep with milk bottle in his mouth, the most common complication is;
a) Dental cries
b) Aspiration pneumonia

315. clavicle fracture in infant:


a) Usually heal without complication
b) Usually associated with nerve injury
c) Need figure of 8

316. Child with mumps. Common complication for this age


a) Meningitis

317. He can sit without assistant, Stand after catching the furniture, say “dada” and uses pincer
grasp, what’s his old?
a) 6 month
b) 8 month
c) 12 month
d) 18 month

318. 2 month old treated with topical erythromycin after conjunctivitis for 7 days, now complaining of
creamy whitish plaque in the tongue, what is the treatment?
a) Oral nystatin
b) Topical Acyclovir
c) Topical Steroid
d) Oral Antibiotics
e) Oral Steriod
 I think if it is candida the answer Oral nystatin but am not sure again!!

319. 7 month old, low grade fever, dry cough, wheezing, hyperinflation with some mild infiltration, what is
the diagnosis?
a) Bronchiolitis
b) Asthma
c) Pneumonia

320. Child brought by the parent with history of 2 days vomiting after assessment it was mild dehydration
what you will give?
a) ORS alone
b) ORS + antiemtic
c) ORS+abx
321. Baby in NICU has a heart rate of 300, good blood pressure level. What should u do:
a) DC shock
b) IV Amiodarone
c) Digoxin
d) Carotid massage

322. Newborn with 300 bpm , with normal BP , normal RR , what do you will do for newborn :
(atrial flutter)
a) Cardiac Cardiversion
b) Verapamil
c) Digoxin
d) Diltzam IV
 If you suspect atrial flutter: Consider digoxin if not already in use because it frequently increases the
conduction ratio and decreases the ventricular rate. , Avoid adrenergic and atropine agents during
sedation or anesthesia for cardioversion. Ketamine is relatively contraindicated

323. Baby with streptococcal pharyngitis :-


a) Treatment after 9 days carries no risk of GN
b) Treatment effective in prevention of GN
c) Clindamycin effective against gram –ve organisms
d) All choices are wrong

324. child with hyperemia and plugging of tympanic membrane had previous history of treated
impetigo so Treatment is:
a) Cefuroxime
b) Amoxicillin
c) Erythromycin
d) Ceftriaxone
e) Cephalexine

325. Typical case of bronchiolitis ( respiratory distress and what not ) you should manage by :
a) Oxygen

326. A child that is sickler and has had recurrent cholecystitis , and found to have 7 gallstones
your management is :
a) Cholecystectomy
b) Uroshydroxelic

327. Only killed vaccine :


a) Hepatitis B

328. Prophylaxis of contact meningitis :


a) Rifampicin for 7 days

329. Mother came with her obese child


a) Decrease calorie intake

330. Best management in case of child with iron overdose ingestion is :


a) Gastric lavage (no defirrox...in choise)

342
331. CPR in child
a) 15 compression and 2 ventilation

332. about varicella vaccine in adult , which is true ;


a) 2 vaccine apart of 1 month
b) 2vaccine apart of 6 month
c) 2 vaccines apart of 2 month
d) 3 vaccine apart of 6 month

333. 8 months child with 3 days fever 40 , vomiting , convulsion , poor feeding & sleep , OE dehydrated ,
depressed anterior fontanel, red ears , no neck stiffness , his 3 year old sibling asymptomatic , which of
the following will give the definitive Dx:
a) CXR
b) CBC with deferential
c) Blood culture
d) CSF analysis
e) Supra-pubic urine analysis

334. At which age child spoke few words


a) 12m
b) 24m

335. Child with low grade fever and congested throat, negative ASO and positive EBV. he has:
a) Infectious mononucleosis
b) URTI
 Explanation:
 The classic criteria for laboratory confirmation of acute infectious mononucleosis include:
1) Lymphocytosis.
2) Presence of at least 10% atypical lymphocytes on peripheral smear.
3) Positive EBV.

336. 12 yrs old complain of LL, UL and face edema and other cardiac sym. Dx :
a) Wet beriberi
b) Dry beriberi
c) Vit. A deficiency
 Explanation:
 Dry beriberi = neurological symptoms.
 Wet beriberi = cardiac symptoms.

337. Child with iron toxicity several hours ago, investigations show iron conc. 700 mg/dl, treated with:
a) Gastric lavage
b) activated charcoal
c) I.V deferoxamine
 Explanation
 Patients with iron toxicity should be treated with IV deferoxime in the following circumstances:
1) Severe symptoms: altered mental status, hemodynamic instability, persistent vomiting, diarrhea.
2) High anion gap metabolic acidosis.
3) Serum iron concentration > 500 mcg/dL.
4) Significant number of pills on X-ray.
134.Child with fever hypotension oliguria
a) Septic shock

135.Child treated by abx develop white patches


a) Antifungal

338. Asthmatic child, how to decrease the allergy:


a) Cover pillow and bed with impermeable material.
b) Throw the rug from house.
c) Wash the clothes and linen in hot water
d) Humidity house with 80 % humidity
e) Cooling clothes

339. 4 y/o child with diarrhea for 2 days is complaining of anal discomfort. Your advice to the mother is:
a) Wash with soap and water after each episode of diarrhea.
b) Wash with cotton in warm water.
c) Put a clean napkin in the underwear.
d) Change the underwear to a highly absorbent diaper

340. Child having vomiting, nystagmus and difficulty in walking the cause is:
a) Dry beriberi
b) Wet Beriberi
c) Pellagra
d) VIT A DEF

341. Infant with sickle cell anemia, what’s true about prophylaxis?
a) Infants should take 23-valent vaccine
b) Children above 2 years take only heptavalent vaccine
c) Even if vaccine taken, if there is contact with ill people child should be given prophylactic Antibiotic
d) If not high risk no need for prophylaxis.

342. 11-year-old patient with sickle cell disease. What is true about pneumococcal vaccine:
a) Not recommended for healthy people.
b) Not necessary for patients who are < 2 years.
c) Give heptavalent after 2 years.
d) Give 23-valent after 2 years.
 All SCD patients < 5 years should receive prophylactic antibiotics (penicillin).
 Regarding pneumococcal vaccine for SCD patients:
 7-valent (heptavalent) “conjugate” pneumococcal vaccine < 2 years old.
 23-valent “polysaccharide” pneumococcal vaccine > 2 years old.

343. Child came with hypertrophic right atrium, what is the congenital anomalies lead to this condition
a) ASD
b) VSD

344. Child with atopic dermatitis at night has stridor plus barking cough on and off from time to time,
diagnosis is:
a) BA
b) Croup
c) Spasmodic Croup
345. A child runs for a long distance then develops pain in the thigh with no redness or tenderness,
best thing to do is:
a) Elevate the leg and cold compression
b) Splint
c) Surgery

346. First sign in increase intracranial pressure:


a) Vomiting
b) Nausea
c) Ipsilateral pupil constrict
d) Contralateral pupil constrict
 Or altered level of consciousness

347. Child with cough, runny nose and fever, O/E: tonsillitis ttt:
a) Paracetamol and throat swab

348. 6 y/o boy present with fever, stridor and O/E showed epiglottitis. Dx:
a) Haemophilus influenza type b
b) Meningoccocus
c) Staphylococcus
d) Streptococcus
 Most common cause of epiglottitis is haemophilus influenza type b

349. 6 month old came with sign and symptom of respiratory distress " fever, tachypnea,
intercostal recession, expiratory wheezing, nasal flare".. best initial management :
a) Oxygen
b) Erythromycin
c) Bronchodilator

350. Picture of child having ulcer near angle of the mouth, bright red in color, 1.5 cm size
a) Fungal infection
b) Impetigo
c) Atopic dermatitis
d) Angular chelitis

351. 9 years old female presented to ER after ingestion almost 20 tablets of OCP and 3 tablets of another
medication. She is clinically stable and there was no signs and symptoms. What will you do:
a) Refer her to gynecologist.
b) Refer her to psychiatrist.
c) Toxicology study
d) No need for intervention.
 In OCP overdose there is no need for intervention if the patient is clinically stable.
352. Newborn 32 week, cyanosed, grunting, flaring of nostrils, the x-ray show diffuse air bronchogram, his
mother is diabetic, what is the diagnosis?
a) Insufficient surfactant
b) Trechoesphgeal fistula
 Respiratory Distress Syndrome (lack of surfactant)
353. Picture, Child with skin lesion at elbow, seen positive wood lamp:
a) Fungal
b) Bacterial

354. Child shows spiral fracture of arm management


a) Refer to orthopedic
b) Open reduction and internal fixation
 Spiral fractures in children raise the suspicion of abuse. They difficult in casting and may require surgery.

355. In paracetamol toxicity:


a) Pencelinemia
b) N-acetylcysteine
c) K intake
d) Deferoxamine

356. 18-months old child brought to you for delayed speech, he can only say "baba, mama" what's
your first step in evaluating him?
a) Physical examination
b) Developmental assessment
c) Head CT
d) Hearing test

357. 2 months infant with severe gastroenteritis, vomiting, diarrhea, increase of the skin turgor,
depressed anterior fontanel, pale, dry mucous membrane, crying but no tears, what is your
management?
a) Aggressive oral rehydration therapy
b) IV saline
c) O.R.S solution given to mother to rehydrate the infant
 Severe dehydration.

358. A child start with waddling gait, what is appropriate investigation:


 Waddling gait at start is normal up to approximately 3 years.

359. Boy with multiple attacks of febrile convulsion what to give him to take at home:
a) Diazepam [rectal gel]
b) Phenobarb

360. 3 year old boy with acute UTI, first thing to do in such acute thing ;
a) Indwelling foley cather drain
b) Voiding cystourethrogram
c) Cystoscopy
 Indication of cystourethrography
 All males with recurrent UTIs (urinary tract infections) or abnormality on ultrasound if first UTI.
 Females < 3 years of age with their first UTI
 Females < 5 years of age with febrile UTIs
 Older females with pyelonephritis or recurrent UTIs
 Suspected obstruction (e.g. bilateral hydronephrosis)
 Suspected bladder trauma or rupture
 Stress incontinence (urine)
361. Child pt with sore throat, ear pain, fever, with nodule, what is organism cause this manifestations:
a) Streptococcus

362. 10 year-old boy with ….to tell that spinal cord length will stop after:
 In children up to L2-L3, In adults up to L1

363. A pediatric patient brought by her parents complaining of vaginal discharge, what is the cause:
a) Foreign body
b) Gonorrhea
c) Trachomatis
 Explanation: Most common cause of vaginal discharge in pediatric patients is FB

364. A Child patient drank something poisonous [Organophosphate] with nausea, vomiting,
diarrhea, hypersalivation, constricted pupil, management:
a) I.V Atropine administration
b) I.V Pralidoxime administration
c) Immediate gastric salvage

365. Which of the following is true regarding German measles (Rubella)


a) Incubation period 3-5 days.
b) It starts with high grade fever in adult only (probably correct check other answers first
 The incubation period for postnatal rubella is 14 to 23 days. The characteristic signs of rubella are
retroauricular, posterior cervical, and posterior occipital lymphadenopathy accompanied by an
erythematous, maculopapular, discrete rash. The rash begins on the face and spreads to the body and
lasts for 3 days. Rose-colored spots on the soft palate, known as Forchheimer spots, develop in 20% of
patients and may appear before the rash.

366. A boy who was bitten by his brother and received tetanus shot 6 month ago and his laceration was 1
cm and you cleaned his wound next you will:
a) Give Augmentin.
b) Suture the wound.
c) Give tetanus shot
d) Send home with close observation and return in 48 hours.
 Since the booster dose has been taken then it is not required to take the tetanus vaccine, however,
if the vaccine hasn’t been taken or 5 years have passed since vaccination a booster dose is
indicated.

367. What is true about marasmus disease:


a) In contrast to kwashoirkor, it affect the low socioeconomic status
b) It is due to late weaning
c) It leads to growth retardation & wt loss

368. Which condition least common associated with endocarditis


a) VSD
b) ASD
c) PDA
d) TOF
369. 10 years old had an episode of rheumatic fever without any defect to the heart. The patient need
to take the antibiotic prophylaxis for how long:
a) 5 months
b) 6 years
c) 15 years

370. After doing CPR on child and the showing asystole:


a) Atropine
b) Adrenaline
c) Lidocaine

371. A mother brought her child with sore throat and barking like cough. His temp. was 38. He
was irritable, with signs of respiratory distress. The diagnosis is:
a) Epiglottitis
b) Croup
c) pneumonia

372. A very very long case about baby 3 months old with 2 days hx of URTI came to you with respiratory
distress and agitated lethargic bilateral ronchi Ix done and he treated with rabivirin what do you
expect the causative organism was:
a) RSV
b) Strept pneumonia
c) H. influenza type B

373. 6 year old with HepAV+ Mother is also positive what vaccination should be given?
 Post-exposure prophylaxis should be considered in individuals who have been exposed to HAV and
have not previously received HAV vaccine. Such patients should receive a single antigen vaccine or IG
(0.02 mL/kg) as soon as possible.

374. Child comes with meningococcal infection, swap obtained and the patient was discharged you
read the culture to find out that it is group A meningitis you call his home to find out that the child is
asymptomatic best step to is:-
a) No action needed.
b) Call the patient.
375. 2 year old child got fever for 2 days then in the 2nd day he became drowsy with vomiting and
diarrhea and appearance of Petechial skin rash which spread rapidly all over the body
a) Measles
b) rocky mountain fever
c) HSP
d) kawasaki
 Initial symptoms:
1) Fever
2) Nausea
3) Emesis (vomiting)
4) Lack of appetite
 Late symptoms
1) abdominal pain
2) Rash
 The most characteristic feature of RMSF is a rash that develops on days 2 to 4 of illness after the onset of
fever, and it is often quite subtle. Younger patients usually develop the rash earlier than older patients.
Most often the rash begins as small, flat, pink, non-itchy spots (macules) on the wrists, forearms, and
ankles. These spots turn pale when pressure is applied and eventually become raised on the skin. The
characteristic red, spotted (PETECHEAL) rash of Rocky Mountain Spotted Fever is usually not seen until
the sixth day or later after onset of the symptom

376. Child with lymphadenopathy splenomegaly one of the following would most likely confirm
the diagnosis,
a) EBV SEROLOGY
b) CBC

377. A child, his family brought him because he is limping and not walking steadily. the baby milestone
was normal according to his age. He had a mild lordotic back Trendelenburg gait Gower sign +ve what is
the best Inx to reach the diagnosis
a) ESR
b) hip Xray or CT
c) m. biopsy

378. 8 years boy BMI = 30 weight and height above 95 percentile , next step :
a) Refer to surgeon
b) Life style modification

379. Before an operation to a child we found him having continues murmur in his right sternal area
what is the next step of management?
a) Postpone and reevaluate the patient again

380. Child came with liver failure; he is not complaining of anything except of yellow discoloration of
skin and now become greenish, This due to:
a) Bilirubin oxidation

381. 15 years old boy with dark urine, dark brown stool, positive occult test, what to do?
a) Isotope scan
b) Abdomen ultrasound
c) X-Ray
d) Barium
382. A 6 years old female from Jizan with hematuria, all the following investigations are needed EXCEPT:
a) HbS.
b) Cystoscopy.
c) Hb electrophoresis.
d) Urine analysis.
e) U/S of the abdomen to see any changes in the glomeruli.
 Cystoscopy is not generally required in children with non-glomerular hematuria. The only indication is a
suspicious bladder mass revealed on ultrasonography

383. Case scenario of child with hx of head trauma who developed hemiparesis, dizziness, loss
of proprioception. Most likely diagnosis:
a) Lobar cerebral hemorrhage

384. Child with meningitis symptoms and no nuchal rigidity, what’s the next diagnostic investigation?
a) CSF

385. Child with positive Gower sign which is most diagnostic test :
a) Muscle biopsy

 Gowers' sign indicates weakness of the proximal muscle of the lower limb. seen in Duchenne
muscular dystrophy & myotonic dystrophy “hereditary diseases”

386. Child with back pain that wake patient from sleep , So diagnosis (incomplete Q)
a) lumber kyphosis
b) Osteoarthritis
c) Juvenile Rheumatoid Arthritis
d) Scoliosis
 JRA or Juvenile Idiopathic Arthritis (JIA) is the most common form of persistent arthritis in children. JIA
may be transient and self-limited or chronic. It differs significantly from arthritis seen in adults. The
disease commonly occurs in children from the ages of 7 to 12

387. 5 years child diagnosed as UTI, what is the best investigation to exclude UTI complication?
a) Kidney US
b) CT
c) MCUG
d) IVU

388. 13 years old child with typical history of nephritic syndrome (present with an urea, cola color
urine, edema, HTN), what is the next step to diagnose?
a) Renal function test
b) Urine sediments microscope
c) US
d) Renal biopsy
351
389. 12 years old female brought by her mother to ER after ingestion of unknown number of Paracetamol
tablets. Clinically she is stable. Blood paracetamol level suggests toxicity. The most appropriate
treatment
a) N-acetylcestine
 IV infusion: 150mg/kg in 200ml D5% over 15mins then 50mg/kg in 500ml D5% over 4hrs then finally
200mg/kg in 1L D5% over 16 hours

390. Child with scrotal swelling, no fever, with a blue dot in the superior posterior aspect of the scrotum
a) Testicular appendix torsion
 Patients with torsion of the appendix testis and appendix epididymis present with acute scrotal pain,
but there are usually no other physical symptoms, and the cremasteric reflex can still be elicited. The
classic finding at physical examination is a small firm nodule that is palpable on the superior aspect of
the testis and exhibits bluish discoloration through the overlying skin; this is called the “blue dot”
sign.
 Approximately 91%–95% of twisted testicular appendices involve the appendix testis and occur
most often in boys 7–14 years old

391. Child with painless hematuria what initial investigation?


a) Repeat urine analysis
b) Renal biopsy
c) Culture

392. Child swallowing battery in the esophagus management:


a) Bronchoscope
b) Insert Foley catheter
c) Observation 12hrs
d) Remove by endoscope

393. child has tracheoesophageal fistula, all can be used in management, except
a) Insertion of chest tube
b) Insertion of NGT
c) Pulmonary toilet
d) Gastrostomy

394. Child with imperforated anus the most useful diagnostic procedure is:
a) Plain abdomen X-ray of with child inverted position
b) Plain X-ray abdomen

395. A child had been bite presented after 18 hour with left arm erythema and itching, what to do?
a) Antihistaminic
b) Oral steroid
c) Subcutaneous epinephrine

396. A baby fell down from stairs and came with multiple contusions some were old and X-ray showed
fracture in radius how to manage:
a) Splinter for his hand
b) Hospitalization and call social worker
 Explanation: Because we are suspecting child abuse.
397. Before 14 d the child was bite, now develop lip swelling & erythema, what type of hypersensitivity?
a) Type 1
b) Type 2
c) Type 3
d) Type 4

398. The preventable nutritional disease in child is:-


a) muscle wasting
b) Kawshoirkor
c) Pica

399. child developed macules progressed to pustular lesions differential diagnosis:


a) Varicalla
b) Herpes

400. Weird scenario about child brought by the parents complains of teeth bleeding after brushing +
gingival bleed + vesicles or papule ... what’s the cause?
a) Candida
b) Herpes
c) Coxacke

401. Child asthmatic and whenever exposed to dust mites, he is having an asthmatic attack. What will
advise his family regarding mites in home?
a) Change the home humidity to 80-85 %
b) wash his clothes and sheets with warm water
c) Cover his pillow with nylon

402. Child came with bilateral swellings in front of both ears. What is the common that could possibly
happen for one within his age?
a) Orchitis
b) Meningitis
c) Encephalitis
d) Epididymitis

403. 5 year old female child with history of pharyngitis for 4 days and persistent odorless vaginal
discharge. Likely etiology:
a) Streptococcus
b) Chlamydia
c) Neisseria Gonorrhea
d) Foreign body
352
404. In rheumatic fever:
a) Bacteria in blood
b) Bacteria lodge in myometrium-not sure but most likely this one right-
c) Skin invasion

405. 6 month baby with mild viral diarrhoea, ttt by ORS as:
a) 100ml/kg for 4 hour then 50 ml/kg /day after
b) 50 ml/kg for 4 hour then 50 ml/kg /day after
c) 100 ml/kg for 4 hour then 100 ml/kg /day after
d) 50 ml/kg for 4 hour then 100 ml/kg /day after
 Deficit Therapy:
 Mild dehydration = 50cc/kg in 4 hrs
 Moderate dehydration = 100cc/kg in 4 hrs
 severe dehydration = IV fluid
 Maintenance Therapy:
 Mild & moderate dehydration = 100 mL/kg/day.
 Severe dehydration = 10-15 mL/kg/hour.

406. The class of antibody responsible for hemolytic disease of the newborn is:
a) IgA
b) IgG
c) IgM
d) IgE
e) IgD

407. A child with hemangioma on the eyelid, in order to prevent amblyopia, operation must be
done within:
a) 1 week.
b) 1 month.

408. A child with polyuria, polydipsia & poor weight gain, random blood sugar >200 mg/dl, fasting
blood sugar >147 mg/dl, this disease is mostly associated with (NB numbers written aren't accurate):
a) HLA-DR4.
b) HLA-DR5.
c) HLA-DR6.
d) HLA-DR7.
 See the 1st aid of the USMLE step 2 CK 7th edition p.114.

409. A young baby brought by his family with an intense rash over his groin region (I forgot the
exact description of the rash), the family tried a large number of medications such as topical
steroids,
?benzoyl something, magnesium something…. With no improvement, what will you give (the
scenario mostly fits with fungal infection):
a) Topical antifungal.
b) Topical steroids.
 See the 1st aid of the USMLE step 2 CK 7th edition p.96 (diaper rash).
410. A young child complains of a pain behind the Lt ear with tenderness, past
Hx of AOM for which he received Abx but didn't complete the ttt, on
examination: tender, redness over the mastoid, otoscope shows congested
TM & loss of cone light reflex, Dx:
a) AOM.
b) Serious OM.
c) Acute mastoiditis.

411. infant of 28 of gestation 900 g go to NICU after resuscitation ABG increase


of Pa CO2 with normal PH next step:
a) Give iv acyclovir
b) Give iv bicarb
c) give iv glucose
 this case is compensated res. acidosis < high pco2 with normal ph >

412. child came with fever , and heterochromia on examination there is enlarged
abdomen , neck swelling
, ptosis:
a) hodgkin lymphoma
b) Neuroblastoma
c) Rabdomyosarcoma
d) wilm's tumor

413. Child UTI with foul smelling urine what is the organism:
a) Proteus mirabilis
b) E.coli
c) klebsiella pneumoniae
d) Streptococuss pneumonia

414. Holding breath attack:


a) Precipitate factor for generalize convulsion
b) Associated with increased incidence of epilepsy in late life
c) Common in children bet 5 and 10 years old

415. child of 7 years came with SOB, cough, he had history of different previous
allergies, on examination he had wheezy chest, what is the most
appropriate initial management:
a) Theophyllin
b) Monteleukast
c) Nebulized albuterol
d) Inhaled corticosteroid

416. 2 Years old Sickle cell patient brought by his adopting parents, he was on
antibiotics due to UTI
(before adoption) and completed the coarse, urine culture is normal..
vital signs T:37.7 What to do?
a) give prophylactic antibiotic
b) Do nothing

417. Child with eczema on hydrocortisone, wt other medication u can add?


a) Tacrolimus

You might also like